ChaseDream

标题: 【每日逻辑练习第二季】【1-9】 [打印本页]

作者: balapupu    时间: 2011-11-27 22:03
标题: 【每日逻辑练习第二季】【1-9】
压力好大,差了6,7,8啊~~~~我要速速补上补上。。。时间不够啊。。。各位一定要坚持。。不要像我一样。。。。

【精练】
9. The five senses have traditionally been viewed as
distinct yet complementary. Each sense is thought to
have its own range of stimuli that are incapable of
stimulating the other senses. However, recent
research has discovered that some people taste a
banana and claim that they are tasting blue, or see a
color and say that it has a specific smell. This shows
that such people, called synesthesiacs, have senses
that do not respect the usual boundaries between the
five recognized senses.
Which one of the following statements, if true, most
seriously weakens the argument?
(A) Synesthesiacs demonstrate a general,
systematic impairment in their ability to use
and understand words.
(B) Recent evidence strongly suggests that there
are other senses besides sight, touch, smell,
hearing, and taste.
(C) The particular ways in which sensory
experiences overlap in synesthesiacs follow a
definite pattern.
(D) The synesthetic phenomenon has been
described in the legends of various cultures.
(E) Synesthesiacs can be temporarily rid of their
synesthetic experiences by the use of drugs.
Weaken Question Problem Set  NO.4




【逻辑链】


33.    (31934-!-item-!-188;#058&005658)


When a caterpillar emerges from the egg on a tree branch, it immediately climbs upward until it finds a leaf bud to eat.  Biologists thought that this behavior displayed an innate tendency to move in the direction opposite to the pull of gravity.  In a recent experiment, a strong light source was placed at the bottom of a tree, and caterpillars, after hatching, climbed downward.


Which of the following hypotheses is best supported by the statements given?


(A) Caterpillars have an innate tendency to move in the direction of gravity.
(B) Newly hatched caterpillars are unable to see in the dark.
(C) Newly hatched caterpillars move towards the strongest light source in the environment.
(D) Newly hatched caterpillars move toward the leaf bud nearest to them.
(E) The eyes of newly hatched caterpillars become less sensitive to light over time.


34.    (31982-!-item-!-188;#058&005681)


For several years, per capita expenditure on prescription drugs in Voronia rose by fifteen percent or more annually.  In order to curb these dramatic increases, the ministry of health prohibited drug manufacturers from raising any of their products' prices.  Even though use of prescription drugs did not expand after this price freeze, per capita expenditure for prescription drugs continued to increase by a substantial percentage each year.


Which of the following, if true, most helps to explain why the ministry's action did not achieve its goal?


(A) After price increases were prohibited, drug manufacturers concentrated on producing new medications to replace existing products.
(B) The population of Voronia rose steadily throughout the period.
(C) Improvements in manufacturing processes enabled drug manufacturers to maintain high profit levels on drugs despite the price freeze.
(D) In addition to imposing a price freeze, the government encouraged doctors to prescribe generic versions of common drugs instead of the more expensive brand-name versions.
(E) After price increases were prohibited, some foreign manufacturers of expensive drugs ceased marketing them in Voronia.


35.    (32076-!-item-!-188;#058&006018)


Which of the following most logically completes the argument?


The last members of a now-extinct species of a European wild deer called the giant deer lived in Ireland about 16,000 years ago.  rehistoric cave paintings in France depict this animal as having a large hump on its back.  Fossils of this animal, however, do not show any hump.  Nevertheless, there is no reason to conclude that the cave paintings are therefore inaccurate in this regard, since __________.


(A) some prehistoric cave paintings in France also depict other animals as having a hump
(B) fossils of the giant deer are much more common in Ireland than in France
(C) animal humps are composed of fatty tissue, which does not fossilize
(D) the cave paintings of the giant deer were painted well before 16,000 years ago
(E) only one currently existing species of deer has any anatomical feature that even remotely resembles a hump


36.    (32124-!-item-!-188;#058&006025)


Outsourcing is the practice of obtaining from an independent supplier a product or service that a company has previously provided for itself.  Vernon, Inc., a small manufacturing company that has in recent years experienced a decline in its profits, plans to boost its profits by outsourcing those parts of its business that independent suppliers can provide at lower cost than Vernon can itself.


Which of the following, if true, most strongly supports the prediction that Vernon's plan will achieve its goal?


(A) Among the parts of its business that Vernon does not plan to outsource are some that require standards of accuracy too high for most independent suppliers to provide at lower cost than Vernon can.
(B) Vernon itself acts as an independent supplier of specialized hardware items to certain manufacturers that formerly made those items themselves.
(C) Relatively few manufacturers that start as independent suppliers have been able to expand their business and become direct competitors of the companies they once supplied.
(D) Vernon plans to select the independent suppliers it will use on the basis of submitted bids.
(E) Attending to certain tasks that Vernon performs relatively inefficiently has taken up much of the time and effort of top managers whose time would have been better spent attending to Vernon's core business.
作者: balapupu    时间: 2011-11-27 22:08
Question #4. June 2001 LSAT, Section #1, #12. The correct answer choice is (A)
The conclusion is in the last sentence, that some people “have senses that do not respect the usual
boundaries between the five recognized senses.” Instead of keeping their senses distinct, these individuals
have an overlap.
Incidentally, the condition discussed in the stimulus is not made up: synesthesiacs (or synesthetes) have a
real condition known as synesthesia. Regardless of that fact, you must find an answer choice that
undermines the conclusion of the argument, something that would suggest their senses do respect the usual
boundaries.
Answer choice (A): This is the correct answer. If the synesthesiacs have a systematic impairment in their
use of language it may not be that their senses overlap but rather that they lack the ability to properly
express themselves. Thus, their claim to taste a banana and see blue might not be a reflection of that
actually occurring but rather a reflection of the words they use to describe taste. If so, this would
undermine the conclusion that the senses of synesthesiacs do overlap. This is a difficult answer to identify
as correct, and less than 50% of test takers are able to do so.
Answer choice (B): The appeal of this answer—and many students keep this as a Contender—is that it
suggests that perhaps other senses are operating, and some test takers make the judgment that these
additional senses account for the sensory overlap in synesthesiacs. Unfortunately, that judgment is not
supported by the answer choice. Not enough information is provided by the answer choice to say what
role, if any, is played by these other senses.
Answer choice (C): This is the most popular wrong answer choice. Do not forget to personalize the
argument and consider how the author would react if faced with this answer. Would he or she surrender
and admit the answer overpowers the argument? Doubtful. The author would probably react to this answer
by saying something along these lines, “Exactly. Since all the individuals are synesthesiacs and suffer from
the same condition, it is not surprising that there would be patterns in the way the senses overlap. Just as
everyone afflicted with emphysema has difficulty breathing, the sensory patterns exhibited by
synesthesiacs are just a product of the condition. The fact that their senses do not follow the usual
boundaries and do so in certain ways is to be expected.” So, instead of surrendering to the answer, the
author would indicate that the answer agrees with the conclusion.
196 The PowerScore LSAT Logical Reasoning Bible
Weaken Problem Set Answer Key
Answer choice (D): This answer is out of the scope of the argument. The “legendary” status of
synesthesiacs does not shed any light on the operation of their five senses.
Answer choice (E): If anything, this may strengthen the argument by indicating that the synesthesiacs are
experiencing some type of phenomenon. Beyond that point, however, no information is given to suggest
that their senses do not respect the usual boundaries.

逻辑链:33. C   34. A  35. C   36. E
作者: xeyyxzty    时间: 2011-11-27 22:12
伤心了~~~做到第三题页面让我刷新了1. background information:The five senses have traditionally been viewed as distinct yet complementary. Each sense is thought to have its own range of stimuli that are incapable of stimulating the other senses.
premise:recent research has discovered that some people taste a banana and claim that they are tasting blue, or see a color and say that it has a specific smell.
conclusion:This shows that such people, called synesthesiacs, have senses that do not respect the usual boundaries between the five recognized senses.
what if the patients have something wrong with their ability to express precisely???
(A) Synesthesiacs demonstrate a general, systematic impairment in their ability to use and understand words.
--right
(B) Recent evidence strongly suggests that there are other senses besides sight, touch, smell, hearing, and taste.
--irrelevant
(C) The particular ways in which sensory experiences overlap in synesthesiacs follow a definite pattern.
--support
(D) The synesthetic phenomenon has been described in the legends of various cultures.
--irrelevant
(E) Synesthesiacs can be temporarily rid of their synesthetic experiences by the use of drugs.
--irrelevant
2.background information:When a caterpillar emerges from the egg on a tree branch, it immediately climbs upward until it finds a leaf bud to eat.  Biologists thought that this behavior displayed an innate tendency to move in the direction opposite to the pull of gravity
premise: In a recent experiment, a strong light source was placed at the bottom of a tree, and caterpillars, after hatching, climbed downward.
conclusion:???
caterpillars climb toward light(A) Caterpillars have an innate tendency to move in the direction of gravity.
--opposite to the fact given
(B) Newly hatched caterpillars are unable to see in the dark.
--irrelevant
(C) Newly hatched caterpillars move towards the strongest light source in the environment.
--right
(D) Newly hatched caterpillars move toward the leaf bud nearest to them.
--not the leaf bud--not nearest to them
(E) The eyes of newly hatched caterpillars become less sensitive to light over time.

--irrelevant
3. background information:For several years, per capita expenditure on prescription drugs in Voronia rose by fifteen percent or more annually.
premise: the ministry of health prohibited drug manufacturers from raising any of their products' prices+???
conclusion: Even though use of prescription drugs did not expand after this price freeze, per capita expenditure for prescription drugs continued to increase by a substantial percentage each year.
the population decreased~~~
(不理解不涨价与销售量的关系...)
(A) After price increases were prohibited, drug manufacturers concentrated on producing new medications to replace existing products.
--irrelevant--right
(B) The population of Voronia rose steadily throughout the period.
-opposite
(C) Improvements in manufacturing processes enabled drug manufacturers to maintain high profit levels on drugs despite the price freeze.
--profits--irrelevant
(D) In addition to imposing a price freeze, the government encouraged doctors to prescribe generic versions of common drugs instead of the more expensive brand-name versions.
--irrelevant
(E) After price increases were prohibited, some foreign manufacturers of expensive drugs ceased marketing them in Voronia.
--use of drugs didn't change--irrelevant
完全不理解~~~4.background information:The last members of a now-extinct species of a European wild deer called the giant deer lived in Ireland about 16,000 years ago.  rehistoric cave paintings in France depict this animal as having a large hump on its back.  Fossils of this animal, however, do not show any hump.
premise:???
conclusion:there is no reason to conclude that the cave paintings are therefore inaccurate in this regard
animals evolve~what if the paintings and fossils are not in the same age~
(A) some prehistoric cave paintings in France also depict other animals as having a hump
--irrelevant
(B) fossils of the giant deer are much more common in Ireland than in France
--irrelevant--the question doesn't doubt whether the paintings are giant deers
(C) animal humps are composed of fatty tissue, which does not fossilize
--right
(D) the cave paintings of the giant deer were painted well before 16,000 years ago
--maybe it has effects on the conclusion, maybe not
(E) only one currently existing species of deer has any anatomical feature that even remotely resembles a hump

--irrelevant
虽然思路游离,但还是选对了~
5. background information:Outsourcing is the practice of obtaining from an independent supplier a product or service that a company has previously provided for itself.  
premise:Vernon, Inc., a small manufacturing company that has in recent years experienced a decline in its profits--outsourcing those parts of its business that independent suppliers can provide at lower cost than Vernon can itself.
conclusion: to boost its profits
outsourcing decreases the costs
(A) Among the parts of its business that Vernon does not plan to outsource are some that require standards of accuracy too high for most independent suppliers to provide at lower cost than Vernon can.
--irrelevant
(B) Vernon itself acts as an independent supplier of specialized hardware items to certain manufacturers that formerly made those items themselves.
--irrelevant
(C) Relatively few manufacturers that start as independent suppliers have been able to expand their business and become direct competitors of the companies they once supplied.
--irrelevant
(D) Vernon plans to select the independent suppliers it will use on the basis of submitted bids.
--irrelevant
(E) Attending to certain tasks that Vernon performs relatively inefficiently has taken up much of the time and effort of top managers whose time would have been better spent attending to Vernon's core business.
--right
作者: 南瓜0729    时间: 2011-11-27 22:12
忘记计时了
P:each of the five senses has its own range of function.
P:but the research shows that some people who taste a banana claim that it is blue.
C: this kind of people have senses that don't respect usual boundry.
P(weaken):THESE people lack sth.
A
A correct
B irrelevant
C ?
D irrelevant
E irrelevant


33.
33s
P:毛虫出生后climbs upward. scientist thinks that it is an innate tendency to climb opposite gravity.
P: scientist places a light on the bottom and 毛虫 climbs downward.
p(假设):毛虫climbs towards the light.
C


34.
37s
P:人均drug expenditure increases.
P:in order to curb these increases, the health minister prohibits them from raising price of drug.
C: though use of drug didn't expand, 人均drug expenditure continues increasing.
P(explain):
D--->A
D opposite
A 政府没有对新药的价格进行限制

35.
37s
P:the last member of gaint deer existed 16,000 years ago.
p:the cave painting showed that it has hump,but the fossil didn't show any hump.
c:there is no reason to conclude that painting is false.
P:hump consist sth that will disappear when it becomes fossil.
C


36.
35s
Butsourcing is obtianing a product from independent supplier.
P:V experienced a decline in its profit.
C:it plans to outsource and independent suppliers can provide lower costs.
P(support):
E


经常自己推想不出来  错了一道题。。。去看看
作者: Jane412    时间: 2011-11-27 22:12
占。。。
作者: qiuhua01234567    时间: 2011-11-27 22:19
标题: 抓狂。。。。。如果我是lawyer多好
占位,哈哈
130S weaken

BackgroundThe five senses have traditionally been viewed as
distinct yet complementary. Each sense is thought to
have its own range of stimuli that are incapable of
stimulating the other senses

Premiserecent research has discovered that some people taste a
banana and claim that they are tasting blue, or see a
color and say that it has a specific smell

ConclusionThis shows that such people, called synesthesiacs, have senses that do not respect the usual boundaries between the five recognized senses

A)Synesthesiacs demonstrate a general,
systematic impairment in their ability to use
and understand words.

----------------------------------------------------------------support
(B) Recent evidence strongly suggests that there
are other senses besides sight, touch, smell,
hearing, and taste.

--------------------------------------------------------------irrelevent
(C) The particular ways in which sensory
experiences overlap in synesthesiacs follow a
definite pattern.

---------------------------------------------------------------correct
(D) The synesthetic phenomenon has been
described in the legends of various cultures.

---------------------------------------------------------------irrelevent
(E) Synesthesiacs can be temporarily rid of their
synesthetic experiences by the use of drugs.

-------------------------------------------------------------------irrelevent

2support 30S

Background: When a caterpillar emerges from the egg on a tree branch, it immediately climbs upward until it finds a leaf bud to eat

Finding: In a recent experiment, a strong light source was placed at the bottom of a tree, and caterpillars, after hatching, climbed downward.

Conclusion:  Biologists thought that this behavior displayed an innate tendency to move in the direction opposite to the pull of gravity.

(A)Caterpillars have an innate tendency to move in the direction of gravity.

------------------------------------------------------------------------------------finding weak this.
(B) Newly hatched caterpillars are unable to see in the dark.

------------------------------------------------------------------------------------no evdience
(C) Newly hatched caterpillars move towards the strongest light source in the environment.

------------------------------------------------------------------------------------correct
(D) Newly hatched caterpillars move toward the leaf bud nearest to them.

------------------------------------------------------------------------------------finding weak this.
(E) The eyes of newly hatched caterpillars become less sensitive to light over time.

--------------------------------------------------------------------------------------no evdience

3explation of argument

Background: For several years, per capita expenditure on prescription drugs in Voronia rose by fifteen percent or more annually In order to curb these dramatic increases, the ministry of health prohibited drug manufacturers from raising any of their products' prices

Premise:  Even though use of prescription drugs did not expand after this price freeze

Conclusion: per capita expenditure for prescription drugs continued to increase by a substantial percentage each year
(A) After price increases were prohibited, drug manufacturers concentrated on producing new medications to replace existing products.

----------------------------------------------------------------------------------------------prescription,not other things
(B) The population of Voronia rose steadily throughout the period.

----------------------------------------------------------------------------------------------irrelevent
(C) Improvements in manufacturing processes enabled drug manufacturers to maintain high profit levels on drugs despite the price freeze.

-----------------------------------------------------------------------------------------------irrelevent
(D) In addition to imposing a price freeze, the government encouraged doctors to prescribe generic versions of common drugs instead of the more expensive brand-name versions.

---------------------------------------------------------------------------------------------------correct
(E) After price increases were prohibited, some foreign manufacturers of expensive drugs ceased marketing them in Voronia.

----------------------------------------------------------------------------------------------------------irrelevent

(A)竟然是(A),我觉得都不对,求助!!

4.22S support

BackgroundThe last members of a now-extinct species of a European wild deer called the giant deer lived in Ireland about 16,000 years ago.  rehistoric cave paintings in France
depict this animal as having a large hump on its back
Premise: Fossils of this animal, however, do not show any hump

Conclusion: there is no reason to conclude that the cave paintings are therefore inaccurate in this regard

(A)some prehistoric cave paintings in France
also depict other animals as having a hump
--------------------------------------------------------------------------------------------------irrelevent
(B) fossils of the giant deer are much more common in Ireland
than in France

---------------------------------------------------------------------------------------------------irrelevent
(C) animal humps are composed of fatty tissue, which does not fossilize

---------------------------------------------------------------------------------------------------correct
(D) the cave paintings of the giant deer were painted well before 16,000 years ago

------------------------------------------------------------------------------------------------------irrelevent
(E) only one currently existing species of deer has any anatomical feature that even remotely resembles a hump

-----------------------------------------------------------------------------------------------------irrelevent

5.25S support

Background: Outsourcing is the practice of obtaining from an independent supplier a product or service that a company has previously provided for itself.  Vernon, Inc., a small manufacturing company that has in recent years experienced a decline in its profits,

Ways: boost its profits by outsourcing those parts of its business that independent suppliers

Conclusion: provide at lower cost than Vernon
can itself
(A)Among the parts of its business that Vernon does not plan to outsource are some that require standards of accuracy too high for most independent suppliers to provide at lower cost than Vernon
can.
-----------------------------------------------------------------------------------------------irrelevent
(B) Vernon

itself acts as an independent supplier of specialized hardware items to certain manufacturers that formerly made those items themselves.
--------------------------------------------------------------------------------------------------irrelevent
(C) Relatively few manufacturers that start as independent suppliers have been able to expand their business and become direct competitors of the companies they once supplied.
(D) Vernon

plans to select the independent suppliers it will use on the basis of submitted bids.
(E) Attending to certain tasks that Vernon performs relatively inefficiently has taken up much of the time and effort of top managers whose time would have been better spent attending to Vernon's core business.
待会又要讨论了

作者: qiuhua01234567    时间: 2011-11-27 22:19
好巧~NO.1
-- by 会员 xeyyxzty (2011/11/27 22:12:19)


你每次都第一个
作者: fox0923    时间: 2011-11-28 03:47
我来占位,然后作昨天的作业~

精练---------------------46s------------------------weaken
B: People's five senses are distinct and complementary.
P: People taste banana but claiming they taste blue, people see color but express smells.
C: These people's senses do not respect the boundaries that these 5 senses share.
Prephrase: These people's 5 senses share boundaries.
Analysis:
(A) Synesthesiacs demonstrate a general,
systematic impairment in their ability to use
and understand words.-----------------------support the conclusion.
(B) Recent evidence strongly suggests that there
are other senses besides sight, touch, smell,
hearing, and taste.---------------------------other senses are irrelevant issue here.
(C) The particular ways in which sensory
experiences overlap in synesthesiacs follow a
definite pattern.-----------------------------contender. If these senses are overlapped, it's possible that people react and express differently when they see and taste.
(D) The synesthetic phenomenon has been
described in the legends of various cultures.-----what??this is idiot choice~totally irrelevant.
(E) Synesthesiacs can be temporarily rid of their
synesthetic experiences by the use of drugs.-----we don't need to know how S. need to be cured.
作者: fox0923    时间: 2011-11-28 04:53
太郁闷了~~咋又错了,受打击了~~

(A) Synesthesiacs demonstrate a general,
systematic impairment in their ability to use
and understand words.

Answer choice (A): This is the correct answer. If the synesthesiacs have a systematic impairment in their
use of language it may not be that their senses overlap but rather that they lack the ability to properly
express themselves. Thus, their claim to taste a banana and see blue might not be a reflection of that
actually occurring but rather a reflection of the words they use to describe taste. If so, this would
undermine the conclusion that the senses of synesthesiacs do overlap. This is a difficult answer to identify
as correct, and less than 50% of test takers are able to do so.----我哭~我就是那<50%的群众~~


(C) The particular ways in which sensory
experiences overlap in synesthesiacs follow a
definite pattern.

Answer choice (C): This is the most popular wrong answer choice. Do not forget to personalize the
argument and consider how the author would react if faced with this answer. Would he or she surrender
and admit the answer overpowers the argument? Doubtful. The author would probably react to this answer
by saying something along these lines, “Exactly. Since all the individuals are synesthesiacs and suffer from
the same condition, it is not surprising that there would be patterns in the way the senses overlap. Just as
everyone afflicted with emphysema has difficulty breathing, the sensory patterns exhibited by
synesthesiacs are just a product of the condition. The fact that their senses do not follow the usual
boundaries and do so in certain ways is to be expected.” So, instead of surrendering to the answer, the
author would indicate that the answer agrees with the conclusion.-----这么比较看的话,C还是不如Aweaken conclusion的程度深,但是我怎么一上来就觉得A irrelevant呢?求解~~~
作者: zz42050524    时间: 2011-11-28 10:09
精练: weaken 36s
The five senses of human are distinct stimuli yet complementary. In an experiment, one people tasted blue and another saw a specific smell.
C: Such people don’t have usual boundaries of the five senses.
Pre: …预测不出来,还是看答案吧
(A) Synesthesiacs demonstrate a general,        只是说S有害,无关
systematic impairment in their ability to use
and understand words.
(B) Recent evidence strongly suggests that there   也是无关结论内容
are other senses besides sight, touch, smell,
hearing, and taste.
(C) The particular ways in which sensory        contender
experiences overlap in synesthesiacs follow a
definite pattern.
(D) The synesthetic phenomenon has been      说这种现象只是传说,和结论无关
described in the legends of various cultures.
(E) Synesthesiacs can be temporarily rid of their  说这种S可以暂时被免除也是无关,答案就是C
synesthetic experiences by the use of drugs.
这~~又错了,,,仔细去看解释
逻辑链
2. must be true 39s
Caterpillars move upward to eat leaf immediately after emerge from eggs.--------C go opposite the direction of gravity.
Caterpillars move down when a bright light is set at the bottom of the tree--------
The direction of C is to the light.
C is the answer.

3. resolve 37s
Per capita of expenditure of drugs has a dramatic increase. Ministry forbid manufactures to raise their prices.
But the expenditure per capita is still increasing even the use has not change.
Pre: It’s not the manufactures raise the prices, rather than someone else.
C: 推论错了,想的太简单了。

4. assumption  28s
A kind of deer has not a large hump, but the ancient painting of the deer illustrate it has.
The fossils of the deer don’t have hump either, since _________
but we can’t say the ancient paintings are fault,

The humps of the deer can’t form into fossils.
C
5.  Support 40s
A manufacture is in low profit now, and it decide to change into outsourcing
Outsourcing would provide lower prices products than making themselves.
Pre: Save the labor cost and the leader will have time to deal with important stuff.
E
作者: zz42050524    时间: 2011-11-28 10:10
太郁闷了~~咋又错了,受打击了~~

(A) Synesthesiacs demonstrate a general,
systematic impairment in their ability to use
and understand words.

Answer choice (A): This is the correct answer. If the synesthesiacs have a systematic impairment in their
use of language it may not be that their senses overlap but rather that they lack the ability to properly
express themselves. Thus, their claim to taste a banana and see blue might not be a reflection of that
actually occurring but rather a reflection of the words they use to describe taste. If so, this would
undermine the conclusion that the senses of synesthesiacs do overlap. This is a difficult answer to identify
as correct, and less than 50% of test takers are able to do so.----我哭~我就是那<50%的群众~~


(C) The particular ways in which sensory
experiences overlap in synesthesiacs follow a
definite pattern.

Answer choice (C): This is the most popular wrong answer choice. Do not forget to personalize the
argument and consider how the author would react if faced with this answer. Would he or she surrender
and admit the answer overpowers the argument? Doubtful. The author would probably react to this answer
by saying something along these lines, “Exactly. Since all the individuals are synesthesiacs and suffer from
the same condition, it is not surprising that there would be patterns in the way the senses overlap. Just as
everyone afflicted with emphysema has difficulty breathing, the sensory patterns exhibited by
synesthesiacs are just a product of the condition. The fact that their senses do not follow the usual
boundaries and do so in certain ways is to be expected.” So, instead of surrendering to the answer, the
author would indicate that the answer agrees with the conclusion.-----这么比较看的话,C还是不如Aweaken conclusion的程度深,但是我怎么一上来就觉得A irrelevant呢?求解~~~
-- by 会员 fox0923 (2011/11/28 4:53:34)

糊糊 我和你的感觉是一样的,我这题我做过了,上来第一下还是把A给排除了,~~~~
作者: fox0923    时间: 2011-11-28 10:54
恩恩~~等待NN解释~~
作者: hexiaomu77    时间: 2011-11-28 13:02
5.support

Background: Outsourcing is the practice of obtaining from an independent supplier a product or service that a company has previously provided for itself.  Vernon, Inc., a small manufacturing company that has in recent years experienced a decline in its profits,

Ways: boost its profits by outsourcing those parts of its business that independent suppliers

Conclusion: provide at lower cost than Vernon itself
话题:vernon外包好
思路:1、外包确实好2、原来不外包的缺点3、外包与profit的关系
(A)Among the parts of its business that Vernon does not plan to outsource are some that require standards of accuracy too high for most independent suppliers to provide at lower cost than Vernon
can.
—————— parts of its business that Vernon does not plan to outsource 是主体无关的讨论范围
(B) Vernon itself acts as an independent supplier of specialized hardware items to certain manufacturers that formerly made those items themselves.
————————Vernon itself acts as an independent supplier of specialized hardware items也是主体无关的范围
(C) Relatively few manufacturers that start as independent suppliers have been able to expand their business and become direct competitors of the companies they once supplied.
———————削弱比较方本是对的,但是削弱方向不对:比较方自身的发展----》应该削弱比较方对于vernon的发展
(D) Vernon
plans to select the independent suppliers it will use on the basis of submitted bids.
—————— Vernon plans to select the independent suppliers话题无关
(E) Attending to certain tasks that Vernon performs relatively inefficiently has taken up much of the time and effort of top managers whose time would have been better spent attending to Vernon's core business.
———————削弱比较法:以前的生产模式不行,改用新的模式后效率会有所提升
作者: winghyy    时间: 2011-11-28 22:19
精炼题:
55s-weaken
Premise: Five senses are distinct and complementary.The research show that some people taste a banana and calim that they are tasting blue, or see a color and say that it has a smell.
Conlcusion: such people have the senses that do obey the boundaries between the 5 senses.
Prephase: The people in the research are suffering from mental disorder.
选B
A) Synesthesiacs demonstrate a general,
systematic impairment in their ability to use
and understand words.——irrelevant
(B) Recent evidence strongly suggests that there
are other senses besides sight, touch, smell,
hearing, and taste.——weaken
(C) The particular ways in which sensory
experiences overlap in synesthesiacs follow a
definite pattern.——wrong, in which way
(D) The synesthetic phenomenon has been
described in the legends of various cultures.——irrelevant (legend)
(E) Synesthesiacs can be temporarily rid of their
synesthetic experiences by the use of drugs.——irrelevant (rid of their synestheic experiences)

又错了。。。。每次最诱惑的答案我都选。
心得:weaken题的正确答案,允许new evidence的出现,所以读清每个答案的意思,不要轻易有关无关。
作者: winghyy    时间: 2011-11-28 22:52
逻辑链
33. 51s-best supported by the argument
P: A caterpillar emerging from the eggs on the tree,  climb upward until it finds a leaf bud to eat. It is believed caterpillar display an innate tendency to move in the direction opposite to the pull of gravity. in a recent experiment, a strong light was placed the at bottom of the tree, the caterpillar climbed dowards.
Prephase: the caterpillar like lights.
选C

34.57s- best explain
P: the expense on prescription drugs has risen by 15% or more annually. In order to curb this increase, the health officer prohibited drug manufactures from raising the price.
Conclusion: Per capiter expense on prescription drugs still rises by a substantial percentage.
Prephase: The inflation or the epidemic disease which needs a lot of prescription drugs.
选A

35. 45s-complete (since)
P: the giant deer was distincted. Prehistoric cave painting depicts it as have a hump on its back.
But the fossils of the giant deer does not show that it has a hump on its back.
C: The depiction of the giant deer is right, since____
Prephase: It is a tradition to paint a hump on the animal in the prehistoric age.
选C

36. 40s- support
V company experienced a decline in profits. So it plants boost its profit by outsourcing by outsourcing some parts of its business that independent suppliers can provide at lower cost than V can.
Prephase: no idea
选D
作者: winghyy    时间: 2011-11-28 22:56
第三题,问影响的是per capita expenditure,不是销售量,per capita expenditure等于price乘以sales,所以价格涨影响per capita expenditure

第五题,D为什么不对。。。求解
outsourcing agency比价,不是有利于cost降低,从而提高利润吗。。。
作者: joelee1991    时间: 2011-11-29 00:09
精炼:23S。
BG:5 senses have been thought seperate and complementary.
P: some people say the banana is blue or ....
c: they call S .and this beyong the boundary.

A
作者: xeyyxzty    时间: 2011-11-29 14:37
第三题,问影响的是per capita expenditure,不是销售量,per capita expenditure等于price乘以sales,所以价格涨影响per capita expenditure

第五题,D为什么不对。。。求解
outsourcing agency比价,不是有利于cost降低,从而提高利润吗。。。
-- by 会员 winghyy (2011/11/28 22:56:46)

第三题我懂了~谢谢


那个D~竞标不都是要最低价吗~没理解你的意思呀
作者: balapupu    时间: 2011-11-29 20:18
P:Five senses are distinct ,they have their own range and can not stimulate other senses?however, one experiment proved that some people taste something and say they see something? this shows that people have S, they have senses do not follow the usual boundries.
Weaken: these people have the problem to speak.
A:
(A) Synesthesiacs demonstrate a general,
systematic impairment in their ability to use
and understand words.-->R(refute the evidence)
(B) Recent evidence strongly suggests that there
are other senses besides sight, touch, smell,
hearing, and taste.-->irrelevant with other senses
(C) The particular ways in which sensory
experiences overlap in synesthesiacs follow a
definite pattern.-->irrelevant with the pattern,
(D) The synesthetic phenomenon has been
described in the legends of various cultures.-->irrelevant with the culture.
(E) Synesthesiacs can be temporarily rid of their
synesthetic experiences by the use of drugs.-->ireelevant with how to get away the S.
作者: ugly5552000    时间: 2011-11-29 21:55
1、Background: the five senses have been viewed as distinct yet complementary.
Premise: recent research discovered that some people tasted a banana and claimed that they were tasting blue or see a color and said that it has a specific smell, this implied that for some people the boundaries among the five senses are not that distinctive.
Prephrase: the five senses works at the same time, and it is reasonable that the five senses of a man who are eating a banana get different de information.
A
A: the answer attacks the very base of the argument, if the synesthesiacs cannot use and understand words as normal people do, their claim will be inconvincible. It weakens the argument.
B: irrelevant
C: what is the definite pattern like? What if the pattern shows indistinctive boundaries of five senses?
D: irrelevant, it cannot attack the argument.
E: if the synesthetic experiences can be rid of by drugs, then it implies the indistinctive boundaries really exist, this supports the argument in some way.
2、Background: when a caterpillar gets out of the egg, it climbs upward on the tree until it finds a leaf bud to eat. This behavior is considered by biologists as an innate tendency to move in the opposite direction of gravity.
Premise: the recent experiment discovers that after placing a strong light source at the bottom of a tree, a caterpillar just coming out of a egg on the tree climbs downwards.
Prephrase: the climbing direction of a caterpillar when it emerges from a egg depends where the adequate light source is.

C
A: incorrect conclusion
B: false conclusion, there is light outside the egg.
C: consistent with the statement
D: what if there is not bud on its way to the light source?
E: if it is true, the newly hatched caterpillars will climb in any direction despite of to the bottom of the tree.
3、Background: the annual expenditure per capital on prescription drugs in Voronia rose by 15% or more. In order to restrain the increase, the authorities kept the drug producers from raising the products’ prices.
Premise: the expenditure per capita still kept rising after implementing the prohibition.
Prephrase: after the prices stop rising, per capita can afford more drugs and more people who can not afford the drugs begin to use the drugs now.

A
A: the policy only aims at the existent drugs, it cannot be imposed on the new drugs, so the new drugs can be set with high price, which cause the rise of expenditure per capita on drugs continue.
B: irrelevant information
C: unconnected with the profit.
D: common drugs is different from prescription drugs
E: if it is true the amount of drugs sold will decline.
4、Background: The prehistoric cave paintings of a now-extinct species of animal is different of the fossils of the animal.
Premise: even the difference exists, the cave paintings should not be regarded as inaccurate.
Prephrase: since some many years passed by, there must be some natural changes on the cave paintings, or because of the natural geographic movement, the fossils might be broke or reconstructed.

C
A: what if the cave paintings depicted animals as having hump are all inaccurate?
B: irrelevant
C: it supports the argument.
D: irrelevant
E: the species in the paintings have extincted.
5、Background: outsourcing means that a company get a product or service which are provided by itself from other independent suppliers.
Premise: in order to curb the decline in profits, Vernon Inc. plans to lower the cost by outsourcing some parts of products which it used to manufacture.
Prephrase: those parts Vernon plan to outsource do account high cost but low profit. The outsourcing suppliers can provide the parts with same quality but lower cost than Vernon do.

E
A: as Vernon cannot find the suitable outsourcing suppliers, it weakens the argument.
B: irrelevant to lower the cost
C: irrelevant
D: what if the bitted suppliers still cannot produce the parts in a lower cost than Vernon does itself?
E: with the help of outsourcing, the top managers would better concentrate in the core business, which is profitable to Vernon.
作者: 风无衣    时间: 2011-11-30 20:45
昨天学校停电断网,补昨天作业~吼吼~
【精炼1-9】
weaken_50s
background information:five senses distinct but complementary. each has its own range of stimuli that incapable of stimulating the other senses
premise:recent survey finds someone can taste banana while taste blue, see a color while has a smell.
conclusion: such people have senses that do not respect the usual boundaries between the five recognized senses.

(A) Synesthesiacs demonstrate a general,
systematic impairment in their ability to use
and understand words.——correct。他因导致。
(B) Recent evidence strongly suggests that there
are other senses besides sight, touch, smell,
hearing, and taste.——irrelevant
(C) The particular ways in which sensory
experiences overlap in synesthesiacs follow a
definite pattern.——have no effect on this issue
(D) The synesthetic phenomenon has been
described in the legends of various cultures.——irrelevant
(E) Synesthesiacs can be temporarily rid of their
synesthetic experiences by the use of drugs.——have no effect on this issue
作者: Rena张    时间: 2012-4-12 22:35
昨天竟然忘记了~~~赶紧今天补上补上!
------------------------------
最近比较忙啊~~~逻辑链的就只好在脑袋里过一遍啦~~就不写出来啦~~~
不太理解第五题为什么选E~~~
1.

Time: 42''

Premise: five senses are distinct yet complementary. A certain kind of people, called XX. Taste a banana, say they are tasting blue.....

Conclusion: such people have sense that do not respect the boundaries between the five sense.

Type: weaken

Prephrase: they are within the boundaries. Yet there must be some other reasons.

Choice: A

2. C   37''

3. B   52''

4. C   21''(这题做过~~)

5. B   49''
------------------------------
发现这一次的逻辑小分队只有俺一个人在做练习诶~~~真是孤军奋战~~~sigh。。。
作者: miffyhui    时间: 2012-4-13 17:01
太郁闷了~~咋又错了,受打击了~~

(A) Synesthesiacs demonstrate a general,
systematic impairment in their ability to use
and understand words.

Answer choice (A): This is the correct answer. If the synesthesiacs have a systematic impairment in their
use of language it may not be that their senses overlap but rather that they lack the ability to properly
express themselves. Thus, their claim to taste a banana and see blue might not be a reflection of that
actually occurring but rather a reflection of the words they use to describe taste. If so, this would
undermine the conclusion that the senses of synesthesiacs do overlap. This is a difficult answer to identify
as correct, and less than 50% of test takers are able to do so.----我哭~我就是那<50%的群众~~


(C) The particular ways in which sensory
experiences overlap in synesthesiacs follow a
definite pattern.

Answer choice (C): This is the most popular wrong answer choice. Do not forget to personalize the
argument and consider how the author would react if faced with this answer. Would he or she surrender
and admit the answer overpowers the argument? Doubtful. The author would probably react to this answer
by saying something along these lines, “Exactly. Since all the individuals are synesthesiacs and suffer from
the same condition, it is not surprising that there would be patterns in the way the senses overlap. Just as
everyone afflicted with emphysema has difficulty breathing, the sensory patterns exhibited by
synesthesiacs are just a product of the condition. The fact that their senses do not follow the usual
boundaries and do so in certain ways is to be expected.” So, instead of surrendering to the answer, the
author would indicate that the answer agrees with the conclusion.-----这么比较看的话,C还是不如Aweaken conclusion的程度深,但是我怎么一上来就觉得A irrelevant呢?求解~~~
-- by 会员 fox0923 (2011/11/28 4:53:34)



对于C项不太理解,能不能解释一下逻辑链
作者: FB小贝    时间: 2012-5-5 23:30
9.【精练】 The five senses have traditionally been viewed as
distinct yet complementary. Each sense is thought to
have its own range of stimuli that are incapable of
stimulating the other senses. However, recent
research has discovered that some people taste a
banana and claim that they are tasting blue, or see a
color and say that it has a specific smell. This shows
that such people, called synesthesiacs, have senses
that do not respect the usual boundaries between the
five recognized senses.
Which one of the following statements, if true, most
seriously weakens the argument?
P1, an experiment , tasting blue, see a smell
C, they have senses that do not respect the usual boundaries between the 5 senses.
W, alternate cause
a. irrelevant b. irrelevant c. correct d. irrelevant e. irrelevant

错了。。。是A。。 求解C的逻辑链
作者: dwindwin1106    时间: 2012-5-10 13:14
(1)P:5 senses have viewed as distinct yet complementary.
P:each sense has its own range of stimuli that are incapable of stimulating the other senses
P:some people (syn) taste banana and claim taste blue, or see a color and say a special smell.
C:syn have senses that do not respect the usual boundaries between the 5 senses.
Weaken: some other reasons cause syn have this kind of special ability
Answer: A
A.Correct ------ 表明是由于语言能力上的损害使得这类人不能正常表达所感受到的,而不是说这类人不遵从usual boundaries
B.Irrelevant ------ 讨论的是syn会有特殊现象的原因,而不是其他sense
C.Strengthen ------ 这类人的感官重叠遵从一定的模式,强化了结论说syn do not repect the usual boundaries
D.Irrelevant ------ 与话题无关
E.Irrelevant ------ 服药后特殊现象消失并不能说明是否syn respect the usual boundaries.

(2)F:when c emerges from the egg, it climbs upward ? B thought this behavior displayed an innate tendency to move in the direction opposite to the pull of gravity
F:a recent study: a strong light source was placed at the bottom of the tree, c climbed downward
must be true
Answer: C
A.Irrelevant
B.irrelevant
C.Correct
D.Irrelevant
E.Irrelevant

(3)F:per capita expenditure on pre drugs in V rose annually
F:prohibit drug manufactures from raising any of their products’ prices
F:use of pre drugs did not expand after this price freeze, but per capita expenditure for pre drugs continued to increase
Resolve:便宜的药没有了,只能买贵的
Answer: A
A.correct
B.Irrelevant
C.Irrelevant
D.Irrelevant
E.Irrelevant

(4)P:the last members of a extinct species of a European wild deer called the giant deer lived in Ireland 16,000 years ago
P:prehistoric cave paintings in F it as having a large hump on its hump.
P:fossils of this animal do not show any hump
C:there is no reason to conclude that the cave paintings are inaccurate, since……
fill in the blanks: the fossil of the hump may be missing?
Answer: C
A.Irrelevant
B.Irrelevant
C.Correct
D.Irrelevant
E.Irrelevant

(5)Putsourcing is the practice of obtaining a product or service that a company has previously provided for itself from an independent supplier.
P:V (experienced a decline in its profits in recent years) plans to boost its profits by outsourcing
Strengthen:?
Answer: C     E
A.Irrelevant
B.Irrelevant
C.Correct   irrelevant
D.Irrelevant
E.Irrelevant   correct


请问有哪位好心人能够解释下最后一题为什么选E吗?始终想的有点纠结,谢谢拉
作者: spencerX    时间: 2012-5-18 22:11
精练
background:
five sense- traditionally- distinct/ complementary
Each sense is thought to have its own range of stimuli that are incapable of stimulating the other senses.
Premise:
recent research show that some people can taste a banana like taste blue(claim)
Conclusion:
people called S- have senses that do not respect the usual boundaries between the five recognized senses.
Pre:
What S feel may be wrong.(claim≠fact)
My answer: A
Analysis:
A.    correct. If these s have some problem in sensing, then what they claim cannot be the evidence that these people have senses that do not respect the usual boundaries between the five recognized senses.
B.    Irrelevant
C.    This may actually support the conclusion
D.    Irrelevant
E.    Irrelevant

逻辑链
1.    fact:
1)    caterpillar tend to climbs upward until it finds a leaf bud to eat when it emerges from the egg (tree branch)—scientist’s explanation: anti- gravity
2)    when there is a light at the bottom of a tree, the caterpillars tend to climbed downward after hatching.
Pre:
caterpillar- tend to climb to- light place
My answer: C
Analysis:
A.    this cannot explain the first phenomenon
B.    this cannot be found in the passage above
C.    Correct. form the passage we see that caterpillars tend to move to the place where is ligher.
D.    this cannot be found in the passage above
E.    this cannot be found in the passage above
2.    
Background: the per capita expenditure for prescription drugs is high and minister want to curb these increase.
paradox:
prohibit drug manufacturers from raising any of their product’s prices& usage doesn’t EXPAND
BUT the per capita expenditure is still high
Pre: population↓
My answer: A
Analysis:
A. I’m not sure whether this answer is correct.
B. if the population rises, then the per capita expenditure should decrease.
C. Irrelevant
D. Irrelevant
E. Irrelevant
有人可以解释一下为什么A对吗?
3.    
Background:The last members of a now-extinct species of a European wild deer called the giant deer lived in Ireland about 16,000 years ago. Prehistoric cave paintings in France depict this animal as having a large hump on its back.
premise one: Fossils of this animal, however, do not show any hump.
Conclusion:there is no reason to conclude that the cave paintings are therefore inaccurate in this regard
Pre: there must be something wrong with the fossils
My answer: C
Analysis:
A.    Irrelevant
B.    Irrelevant
C.    Correct. if the hump cannot be persevered in the fossil, then the cave paintings are not wrong
D.    Irrelevant
E.    Irrelevant
4.    Background: Outsourcing is the practice of obtaining from an independent supplier a product or service that a company has previously provided for itself
Premise: V- outsourcing some parts of it business to independent suppliers can provide at lower cost than V
Conclusion: profit↑
My answer:
A.    Irrelevant
B.    Irrelevant
C.    Correct. if these companies once become the competitor of V, then they may be cannot achieve as much profit as they do before.(是few啊,是否定啊,这个是opposite答案)
D.    Irrelevant
E.    Irrelevant
作者: spencerX    时间: 2012-5-19 21:17
请问有哪位好心人能够解释下最后一题为什么选E吗?始终想的有点纠结,谢谢拉
-- by 会员 dwindwin1106 (2012/5/10 13:14:57)




我现在觉得是,本来一些琐细的事情将要占用top manager很多处理核心问题的时间,但是现在如果外包了,那么这些时间久可以节约下来,那么说不定就可以更好地处理core problem,这样profit就会上升了(因为时间问题,公司的工作是否efficient,都是cost之一啊,节约时间应该就算节约cost了吧)
作者: 卖红薯a    时间: 2012-5-22 23:16
【精练】
9.
1)计时:42s
2)逻辑链Situation
Background information: 5 senses have been viewed as distinct yet complementary. Each sense has its own range of stimuli that is not able to stimulate other senses.
Premise: study show: some ppl taste a banana and claim that they are tasting blue, or see a color say that it has a special smell
Conclusion: such ppl have senses that do not respect the usual boundaries between the 5 senses
3)推测: do respect the usual senses
4)选项分析:选C
A. Irrelevant. words using/ understanding ability
B. Support the conclusion.
C. Right. 排除法作的
D. Irrelevant. culture included or not
E. Irrelevant. remove their synesthetic experiences

【逻辑链】
33.
1)计时:37s
2)逻辑链Situation
Background information: a caterpillar climbs upward until it finds a leaf bud to eat after emerging
Premise: biologist thought: it moves reversely to the pull of gravity
Conclusion: an experiment: a strong light was placed at the bottom of a tree, and caterpillars climbed downward after hatching
3)推测: C’s moving direction is influenced by gravity
4)选项分析:选A
A. Right.
B. Undermine the statement.
C. Support, but not the best choice
D. Does not support the statement
E. Does not support the statement

34.
1)计时:36s
2)逻辑链Situation
Background information: per capita cost on drugs rose↑15% or more
Premise: in order to curb the increase, ministry prohibit drug producers from raising the drug’s price
Conclusion: after price freezing: the use of drug didn’t expand, but per capita cost still increase↑greatly
3)推测:
4)选项分析:选E
A. doesn’t explain why it didn’t achieve the goal. New medicine-- > drug’s price shouldn’t↑(思考:因为制造商不多制造此种药,这种药在市场上的需求持续上升,导致其价格持续增长)
B. Opposite. More ppl need, but the use didn’t expand
C. Irrelevant. producer’s profit
D. doesn’t explain why
E. Right.

35.
1)计时:37s
2)逻辑链Situation
Background information: giant dear who lived in Ireland 16,000 years ago is now extinct.
Premise: cave paintings in France: this animal has a large hump on its back,
but fossils do not show humps
Conclusion: no reason to conclude that the cave paintings are inaccurate
3)推测: since the fossils of hump maybe disappear, maybe the animal of the painting is not GD
4)选项分析:选D
A. Irrelevant. other animals
B. not enough info
C. can’t explain
D. Right.
E. Irrelevant.

36.
1)计时:48s
2)逻辑链Situation
Background information: outsourcing is the practice of obtaining from an independent supplier a product or service that a company used to provide for itself.
Premise: V experienced a decline in its profits recent years,
Conclusion: V plans to boost its profit by outsourcing those parts of its business that independent suppliers can provide at lower cost than V can.
3)选项分析:选E
A. Opposite.
B. Irrelevant.
C. Opposite.
D. Opposite.
E. Right.
作者: baseboss    时间: 2012-5-23 19:53
1. Five sense seems do not have the boundaries among each other.
presume:
1.mental feeling can not instead the function of each sence.

A) Synesthesiacs demonstrate a general,
systematic impairment in their ability to use
and understand words.
-->irrevelant
(B) Recent evidence strongly suggests that there
are other senses besides sight, touch, smell,
hearing, and taste.
-->irrelevant
(C) The particular ways in which sensory
experiences overlap in synesthesiacs follow a
definite pattern.
-->irrelevant
(D) The synesthetic phenomenon has been
described in the legends of various cultures.
-->irrelevant
(E) Synesthesiacs can be temporarily rid of their
synesthetic experiences by the use of drugs.
-->irrelevant

选C

2.Normally,caterpilla climb toward the opposite of the gravity after coming out of egg.
 It change the direction to the strong light when stimuli by it.
presume:
Caterpilla has the ability to sensitive the light.
选C

3.Ministry want to curb the increasing the comsumption of drug by prohibite the sales price of drug incerase.
presume
1 Doctor will give more drug for patient than before.
2 Patient will buy drugs for themselves than before.
A

4.Paint for the animal has hump on the back but can not see from the fossile.
presume:
No bone on the hump area.
C

5 V want to outsoucing parts to have more profit.
presume:
Specially process and lager manufaction level can reduce the cost.
D
作者: 二楼往下掉    时间: 2012-6-4 01:38
1.
(1)41s weaken
(2)逻辑链
Background:
Traditionally, people believe that five senses are distinct and complementary.
Primes:
Recent research shows that some people when eat banana claim that they are tasting blue
Conclusion:
S have senses that do not have the boundaries between them.
(3)不能确定他们确实看到了啥或者闻到了啥,这些都只是他们的主观感受。
(4)A:S现象是一种对于他们运用词汇理解词汇的能力的损伤,与本文讨论的这种现象的存在性无关,loser——语言表达的有损伤,表明这些感官并不重叠
B:文中讨论的是这几种感觉之中有没有交叉,不考虑可能存在的其他感觉,loser
C:他们的特殊感觉都遵循已经确定了的一种模式,所以他们的感觉并不是个性化的,所以这可能并不是S所描述的那样,contender——举例这种同质的表现是完全正常的
D:文中讨论的是S这种现象的存在性,S现象被如何描述与本文讨论的中心无关,loser
E:文中在讨论这种现象的存在性,在基于这种现象存在的基础上的讨论都与文章中心无关,loser
所以,选C

2.
(1)27s supported by—can be true
(2)逻辑链
Background:
Caterpillar after hatch would immediately climb upward until meet the first leaf bud.
Primes:
The caterpillar climbs to against the gravity.
Put the light at the bottom of the tree, and the caterpillar climb downward.
(3)I guess:Caterpillar climb trend to light
(4)A:在文章的前半部分提到,但是后半部分又提出了一个新的观点,所以loser
B:文中没有提到caterpillar能不能在黑暗中看到东西,只是说caterpillar会向着光爬,loser
C:文中的实验现象是:caterpillar一开始向上爬,后来在树底放了光线,caterpillar就向下爬了,Contender
D:文中从来没这么说过,loser
E:能向着光爬,怎么会对光不敏感,loser

3.
(1)1’ 20’’  explain
(2)逻辑链
Background:
Per capita expenditure on prescription drugs in Voronia rose by fifteen percent or more annually
Plan:
The ministry of health prohibited drug manufacturers from raising any of their products' prices
Goal:
Curb these dramatic increases
Result:
Even though use of prescription drugs did not expand after this price freeze, per capita expenditure for prescription drugs continued to increase by a substantial percentage each year
(3)其他drug相关的费用在增长,例如运费等等
(4)A:研究新药代替旧药,但文中并未给出新药与旧药的价格对比,所以有办法判断对人均消费的影响,loser
B:V的人口稳步增长,那应该人均花费变少,loser
C:提到企业profit但是与人均花费无关,loser
D:用普通的药代替昂贵的P种药,人均消费应该会降低,loser
E:外国企业撤出市场,导致本国药品数量稀缺,因此价格增长,contender
我的答案:E
正确答案:A

4.
(1)35s
(2)逻辑链
Background:
The last members of a now-extinct species of a European wild deer called the giant deer lived in Ireland about 16,000 years ago.
Primes:
<1>Cave painting depict this animal had large hump on its back
<2>Fossils show this animal had no hump on its back
<3>?
Conclusion:
No reason to conclude the cave painting is inaccurate
(3)还有东西能证明painting是对的
Fossil不能确定是不是那种deer的
(4)A:都有hump更不可信吧,loser
B:化石多少不重要,关键是化石能不能显示出是否有hump,loser
C:hump永远都不会在fossil中出现,所以有没有hump fossil说了不算,contender
D:16,000年前,但是不一定是这种deer活着的年代,所以对argument没有影响,loser
E:与现在deer是否有hump无关,不能说明那个地方的那个品种的deer就一定有或没有hump,loser
所以,选C

5.
(1)30s support
(2)逻辑链
Background:
The last members of a now-extinct species of a European wild deer called the giant deer lived in Ireland about 16,000 years ago.
Vernon, Inc., a small manufacturing company that has in recent years experienced a decline in its profits
Plan:
By outsourcing those parts of its business that independent suppliers can provide at lower cost than Vernon can itself.
Goal:
Boost its profits
(3)原来生产某一部分的成分太高,可以讲前后成本作比较
可以减少员工,降低企业运行成本
(4)A:not plan to outsource,讨论范围转移,loser
B:与自己将部分业务outsourcing无关,不应该讨论自己承接的别的企业的outsourcing,loser
C:outsourcing使得企业与相关行业的企业有了可以竞争的实力,contender
D:与V要选择的企业无关,loser
E:非核心可以outsourcing的业务会占用manager很多用来关心core business的时间,contender
所以,B虽然说企业会在面对相关企业更有竞争力,但是表达的不够明确,所以选E
作者: emmasy    时间: 2012-6-8 10:37
1.
事实:Each sense is thought to have its own range of stimuli that are incapable of stimulating the other senses. Some people can.
结论:these people have senses that do not respect the usual boundaries between the five recognized senses.
a.weaken conclusion
b.support
c.support
d.irrevelant
e.irrevelant
作者: emmasy    时间: 2012-6-8 10:58
2.
事实:the action that after hatching it immediately climbs upward until it finds a leaf bud to eat is thought to be  an innate tendency to move in the direction opposite to the pull of gravity. A new experiment, when a strong light source was placed at the bottom of a tree, and caterpillars, after hatching, climbed downward.
a.include 1, not 2
b.irrevelant
c.include 2, not 1
d.irrevelant
e.conclusion include both

3.
事实:per capital expenditure on drugs is increasing. However, the drugs prices remain the same, and the use of drugs remain the same as well.
a.explain
b.irrevelant
c.irrevelant
d.irrevelant
e.irrevelant

4.做太熟了

5.
事实:the company plans to let other company provide them with products at lower price than those previous produced by itself.
结论:the company can boost its profit
a.irrevelant
b.irrevelant
c.irrevelant
d.irrevelant
e.support
作者: jetyxo    时间: 2012-6-12 22:52
1   premise1: five senses are distinct,they have their own ranges and can't stimulate other senses
    premise 2: experiments: some people eat banbana--->blue; see a  color--->smell
    conclusion:such people have senses that don't respect the rule of boundary between the five senses
  weaken: have no idea
    A B D E irrelevant  ---> C (wrong)  the right answer :A   weaken the premise
     
2  caterpillar  no light--->climb up to the leaf--->climb agnist the gravity
                 a light at the bottom---> climb down
   support:  caterpillar climb to the light---.answer  C

3   to turb expenditure on  prescription drugs --->prohibited drug manufacturers from raising any of their products' prices
    effect: uesage of prescription drugs didn't expand,but the expenditure continue increase
    reason?  A

4   prehistorical  cave painting depict giant deer have a bump,however, fossils of giant beer don't have the bump
    noreason to conclude the cave painting were inaccurate ,reason?---> the bump of the beer can't fossilize C
    fossils of beer=/  beer

5   outsourcing can make the cost lower, to increase profits,the company should choose outsourcing
    support: outsourcing doesn't have bad effects on the profits of the company
     A B C D irrelevant  the answer: E
作者: LuckyYolandaLi    时间: 2012-6-23 13:53
1.

1)计时:54s

2)逻辑链:

Background information: The five senses have traditionally been viewed as distinct yet complementary. Each sense is thought to have its own range of stimuli that are incapable of stimulating the other senses.

Premise: some people taste a banana and claim that they are tasting blue, or see a color and say that it has a specific smell.

Conclusion: people, called synesthesiacs, have senses that do not respect the usual boundaries between the five recognized senses.

3)推测:People, such as those who taste a banana and claim that they are tasting blue were applying a kind of rhetoric in their works.

4)选项分析:选A

B: Irrelevant.

E: Irrelevant.



2.

1)计时:52s

2)逻辑链:

Background information: When a caterpillar emerges from the egg on a tree branch, it immediately climbs upward until it finds a leaf bud to eat.

Premise: Biologists thought that this behavior displayed an innate tendency to move in the direction opposite to the pull of gravity.

Conclusion: In a recent experiment, a strong light source was placed at the bottom of a tree, and caterpillars, after hatching, climbed downward.

3)推测:。。。

4)选项分析:选A(正确答案C

(提议理解错误,应该是support,我以为是support



3.

1)计时:54s

2)逻辑链:

Background information: For several years, per capita expenditure on prescription drugs in Voronia rose by fifteen percent or more annually.

Premise: For several years, per capita expenditure on prescription drugs in Voronia rose by fifteen percent or more annually.Use of prescription drugs did not expand after this price freeze

Conclusion: Per capita expenditure for prescription drugs continued to increase by a substantial percentage each year.

3)推测:

4)选项分析:选A

A: 这个我不太明白,但两外几个选项我感觉都不对,所以就选这个了吧。。。

B: As the population rose, the total expenditure for prescription drugs may increase. But the per capita expenditure cannot be affected.

C: The per capita expenditure for prescription drugs cannot be directly influenced by the profit levels, and it will remain unchanged as long as price remains unchanged.

D: According to this option, it will receive the converse outcome against the one in the argument above.

E: 效果同D



4.

1)计时:55s

2)逻辑链:

Background information: The last members of a now-extinct species of a European wild deer called the giant deer lived in Ireland about 16,000 years ago.

Premise: Prehistoric cave paintings in France depict this animal as having a large hump on its back.  Fossils of this animal, however, do not show any hump. Since______.

Conclusion: There is no reason to conclude that the cave paintings are therefore inaccurate in this regard.

3)推测:Maybe the hump was taken away by humans, so the fossils don’t show it.

4)选项分析:选C

A: Converse to the conclusion.

B: Irrelevant.

C: Correct.

D: Irrelevant.

E: 有点irrelevant, 还有点Converse to the conclusion.



5.

1)计时:55s

2)逻辑链:

Background information: Outsourcing is the practice of obtaining from an independent supplier a product or service that a company has previously provided for itself.

Premise: Vernon, Inc., a small manufacturing company that has in recent years experienced a decline in its profits.

Conclusion: Vernon plans to boost its profits by outsourcing those parts of its business that independent suppliers can provide at lower cost than Vernon can itself.

3)推测:The parts of its business that independent suppliers can provide are as good as Vernon can itself.

4)选项分析:选C(正确答案E

A: Irrelevant.

B: Irrelevant.

C: Correct.

D: Irrelevant.

E: 这个我感觉也有点对。。。

CE二选一,到底还是选错了。。。不太明白)
作者: jiajiajudy    时间: 2012-6-26 04:49
1.    BG: five senses, distinct and complementary. Each has its range of stimuli, not stimulate the other
Counter premise: research has discovered taste banana claim blue? conclusion: syn, not respect usual boundaries
(A)Synesthesiacs demonstrate a general, systematic impairment in their ability to use and understand words.
(B) irrelevant
(C) irrelevant
(D) irrelevant
(E) irrelevant
2.     BG: caterpillar emerges from egg, climbs upward until it finds a leaf bud
Premise: biologist: displayed an innate tendency to move in the direction opposte ofgravity
Counter premise: strong light source was placed at the bottom of tree, caterpillars climbed downward
(A) paradox
(B) not effect
(C) Newly hatched caterpillars move towards the strongest light source in the environment.
(D) not effect
(E) not effect

3. BG: per capita expenditure on prescription drugs rose by 50% or more
Premise: prohibited drug manufacturers from raising prices
Conclusion: per capita continued to increase
(A)irrelevant
(B) The population of Voronia rose steadily throughout the period.
(C) irrelevant
(D) irrelevant
(E) not effect
4. BG: Giant deer, the  last members of a now-extinct pecies of E wild dear in iraland
Premise: cave painting shows that this  animals as having a large hump
Counter premise: fossils do not show any hump, france
Premise: there is no reason to conclude that the cave painting are inaccurate
(A) irrelevant
(B) irrelevant
(C) animal humps are composed of fatty tissue, which does not fossilize
(D) irrelevant
(E) irrelevant
5. BG: outsourcing
Premise: V profits decline, boost by outsourcing? lower cost
(A) irrelevant
(B) irrelevant
(C) weaken
(D)irrelevant
(E) Attending to certain tasks that Vernon performs relatively inefficiently has taken up much of the time and effort of top managers whose time would have been better spent attending to Vernon's core business.
作者: jiajiajudy    时间: 2012-6-26 04:50
那个prescription drug A 选项真的不太明白
作者: angelafeng    时间: 2012-6-26 13:38
【精练】
9.38s

Background informationThe five senses have traditionally been viewed as distinct yet complementary. Each sense is thought to have its own range of stimuli that are incapable of stimulating the other senses.

Premise recent research has discovered that some people taste a banana and claim that they are tasting blue, or see a color and say that it has a specific smell.

ConclusionThis shows that such people, called synesthesiacs, have senses that do not respect the usual boundaries between the five recognized senses.

Prephrase:(weakenexist the 6th sense



(A) Synesthesiacs demonstrate a general,
systematic impairment in their ability to use
and understand words.------------this has no relation with the five senses.---------If the synesthesiacs have a systematic impairment in their use of language it may not be that their senses overlap but rather that they lack the ability to properly express themselves.
(B)
Recent evidence strongly suggests that there
are other senses besides sight, touch, smell,
hearing, and taste.-------------correct-------------Not enough information is provided by the answer choice to say what
role, if any, is played by these other senses.
(C) The particular ways in which sensory
experiences overlap in synesthesiacs follow a
definite pattern.-------------irrelevant
(D) The synesthetic phenomenon has been
described in the legends of various cultures.------------irrelevant
(E) Synesthesiacs can be temporarily rid of their
synesthetic experiences by the use of drugs.-----------this just help to explain how to cure Synesthesiacs.

【逻辑链】
33.    (31934-!-item-!-188;#058&005658)
37s

Background informationWhen a caterpillar emerges from the egg on a tree branch, it immediately climbs upward until it finds a leaf bud to eat.  

PremiseIn a recent experiment, a strong light source was placed at the bottom of a tree, and caterpillars, after hatching, climbed downward.  

ConclusionBiologists thought that this behavior displayed an innate tendency to move in the direction opposite to the pull of gravity.
prephrase
:(support
answer
C

34.    (31982-!-item-!-188;#058&005681)
33S

Background informationFor several years, per capita expenditure on prescription drugs in Voronia rose by fifteen percent or more annually.  

PremiseIn order to curb these dramatic increases, the ministry of health prohibited drug manufacturers from raising any of their products' prices.   use of prescription drugs did not expand after this price freeze.

Conclusionper capita expenditure for prescription drugs continued to increase by a substantial percentage each year.
prephrase: (explain)
一些不受政府控制的外国药价格更高了
answer:E
(A) After price increases were prohibited, drug manufacturers concentrated on producing new medications to replace existing products.
(B) The population of Voronia rose steadily throughout the period.
(C) Improvements in manufacturing processes enabled drug manufacturers to maintain high profit levels on drugs despite the price freeze.
(D) In addition to imposing a price freeze, the government encouraged doctors to prescribe generic versions of common drugs instead of the more expensive brand-name versions.
(E) After price increases were prohibited, some foreign manufacturers of expensive drugs ceased marketing them in Voronia.------------ use of prescription drugs did not expand

35.    (32076-!-item-!-188;#058&006018)
29s
background information
The last members of a now-extinct species of a European wild deer called the giant deer lived in Ireland about 16,000 years ago.  rehistoric cave paintings in France depict this animal as having a large hump on its back.  

PremiseFossils of this animal, however, do not show any hump.  

Conclusionthere is no reason to conclude that the cave paintings are therefore inaccurate in this regard
prephrase
:(explain the animal’ s hump cannot become fossils.

Anwer:C

36.    (32124-!-item-!-188;#058&006025)
34S

Background informationOutsourcing is the practice of obtaining from an independent supplier a product or service that a company has previously provided for itself.  

PremiseVernon, Inc., a small manufacturing company that has in recent years experienced a decline in its profits, outsourcing those parts of its business that independent suppliers can provide at lower cost than Vernon can itself.
conclusion
boost its profits
prephrase
:(supportin the future, the independent suppliers cannot become competitors of Vernon.
answer
E

(A) Among the parts of its business that Vernon does not plan to outsource are some that require standards of accuracy too high for most independent suppliers to provide at lower cost than Vernon can.
(B) Vernon itself acts as an independent supplier of specialized hardware items to certain manufacturers that formerly made those items themselves.
(C) Relatively few manufacturers that start as independent suppliers have been able to expand their business and become direct competitors of the companies they once supplied.
(D) Vernon plans to select the independent suppliers it will use on the basis of submitted bids.
(E)Attending to certain tasks that Vernon performs relatively inefficiently has taken up much of the time and effort of top managers whose time would have been better spent attending to Vernon's core business.

作者: 呵呵牙    时间: 2012-6-28 10:57
1. 32s
Background Information: The five senses have been viewed as distinct. Each sense is thought to have its own range of stimuli.
Premise: Some people taste a banana and claim that they are tasting blue, or see a color and say that it has specific smell.
Conclusion: Synesthesiacs have senses that do not respect the usual boundaries between the five recognized senses.
Prephrase: People just follow usual rules but sometimes they can describe different feeling by five senses.
选A
B (other senses) ——support
C (definite pattern) ——irrelevant
D (legends of various cultures)——irrelevant
E (drugs)——irrelevant

2. 34s
Background Information: A caterpillar appears and climbs upward until it finds a leaf bud to eat.
Premise: A strong light source was placed at the bottom of a tree, and caterpillars, after hatching, climbed downward.
Conclusion: This behavior displayed an innate tendency to move in the direction opposite to the pull of gravity.
Prephrase: caterpillars followed light to move.
选C

3. 38s
Background Information: Per capita expenditure on prescription drugs in Voronia rose by 15% or more annually.
Premise: The ministry of health prohibited drug manufacturers from raising products’ price to curb increases.
Conclusion: Per capita expenditure on prescription drugs continues to increase each year.
Prephrase: More and more people use prescription drugs and the price can be control that people can afford it.
选A

4. 55s
Background Information: Now-extinct species of European wild deer called the giant deer lived in Ireland about 16,000 years ago.
Premise: Prehistoric cave paintings in France depict this animal as having a large hump on its back, but fossils do not show any hump.
Conclusion: There is no reason to conclude that the cave paintings are inaccurate.
Prephrase: The paintings just describe people’s thought or the time of cave painting is not correct.
选D ——————correct answer is C

5. 40s(有点不太明白)
Background Information: Outsourcing is the practice of obtaining from an independent supplier a product or service that a company has previously provided for itself.
Premise: Vernon, Inc. has decline in its profits.
Conclusion: This company plans to boost its profits by outsourcing that suppliers can provide at lower cost.
选E
作者: cleotina    时间: 2012-7-1 19:33
46'
weaken
B: people have 5 senses, and each of them are not stimulating others, some

one taste a banana and say it is in the color of blue.
C: this kind of people do not respect the traditional five senses boundary
Guess:
1, this people are psycosis
2, ther is no other clues to help them certain the irrelative sense
3, this people may have a conception confuse about the color
OA: A
B: strong
others: I

38''
conclusion
B: Caterpillar emerges from eggs will immediatelly clime up until find a

leaf to eat, this phenominon seems like a innate behavior to go at the

opposite position of the pull of the gravity.
research: when set a strong light source in the ground, the caterpillar

climbs downward
Guess:
1 caterpillar climbs toward the direction of the light
2 if the light is strong enough, will change the habit to climb upward
3 the carerpillar clim toward the strong light
OA: C
A&D: wrong
B&E: I

35''
explain
B: For sevaral years, percapital expenditure on the medicine keep increasing, in order to stop this increase, the pertaining commission freeze the price.
Result: the trend of increase has not change.
guess: other factors to prove the consumption: doctor tend to increase the prescrition to earn more money; the physic body is not as health as before
OA: A  drug industry try to produce new drugs
E:weaken
Others: wrong

35''
reason
B: a extincted deer, found in one cave, was depicted with a large hump on the back. However, the fossils of the deer shows no domp on the back.
C: the picture in the cave not wrong
Guess: the Hump can not formed into fossil stytle
OA: C
AB: wrong
DE:I

45''
support
B: outsourcing is the practice of obtaining from a part of source which supported by themselves before.
R: a company will make some of his source into the outsourcing one to improve its profits
Guess: it did will decrease the cost and not affect the production
OA: E
ad:I
AB:wrong
作者: Donts    时间: 2012-7-15 11:37
1.计时41''premise:5 senses are distinct but complementary.people eating banana may see blue colour and a special smell.conclusion:those people have senses that do not respect the conventional boundaries.
推测 :不是可以complementary吗,可能是它在起作用才导致上述表现的。
(A) Synesthesiacs demonstrate a general,
systematic impairment in their ability to use
and understand words. use和undstand words和感官有关吗?无关吧,好像和大脑有关,起码和例子中的感官无关。
(B) Recent evidence strongly suggests that there
are other senses besides sight, touch, smell,
hearing, and taste.那和synesthesiacs好像无关的样子。
(C) The particular ways in which sensory
experiences overlap in synesthesiacs follow a
definite pattern.并没有越界,只是overlap。正确,不过感觉怪怪的。
(D) The synesthetic phenomenon has been
described in the legends of various cultures.并没有削弱,无关。
(E) Synesthesiacs can be temporarily rid of their
synesthetic experiences by the use of drugs.题目中也没提到drug,无关。
订正:A

总结:思路完全扭曲啊,我还YY地很开心= =word居然是use of language的问题。所以不是他们感官出了问题,是他们的语言能力出了问题。完全想不到啊想不到TAT。遇上这种正确答案诡异,错误答案迷惑人的情况,我也只能掩面泪奔了吧。
作者: showysimple    时间: 2012-7-30 08:13
9. 瞢对
[Background] - five senses are different and can’t stimulate the others.
[Premise] – HOWEVER, some ppl taste a banana and claim that they are tasting blue, or see a color and say that it has a specific smell.
[Conclusion] – synesthesiacs do not respect the usual boundaries between senses

求weaken
[Prephrase] –?

(A)
(B) irrelevant/indirect-题干讨论的是five senses, 其它sense与其无关。看了bala的解释后补充:选项点到了‘other senses’,但没有直接讲清楚‘other senses’与‘synesthesia’的关系。
(C) irrelevant, 解题的方向应是“为什么synesthesiacs的共感不等于感官混乱”,但(C)选项只是更为展开描述synesthesia pattern (共感)的情况,并没有解释‘为什么’。
(D) out of scope
(E) strengthen/irrelevant
作者: showysimple    时间: 2012-7-30 08:15
33.    (31934-!-item-!-188;#058&005658), 作对

[Premise]
1.    caterpillar climbs upward a tree after emerging from the egg → hypo by bio-ist that caterpillar move in the direction opposite to the gravity
2.    caterpillar climbs downward after a strong light at the bottom of a tree

求[conclusion]

(A) 因果关系成立于premise 2, 不成立与premise 1
(B) out of scope
(C) right answer
(D) 因果关系成立于premise 1, 不成立于premise 2
(E)  与premise 2相反,如果感光弱怎么会往下迎着光线爬

34.    (31982-!-item-!-188;#058&005681),做错

[Background] – per capital expend. (on prescription drugs in V) rose 15%+.
[Premise]
1.    ALL drug prices are prohibited to rise
2.    Use of prescription drugs didn’t expand
[Conclusion] – per capital expend. continue to increase

求explain
[Prephrase] –?

Per capital expend. = total expend. (price x use of pscpt.drug) / population
(A) 用排除法的,看到有人说“政府没有限定新药的价钱”,还需消化
(B) 根据以上公式,人数增加的话,人均应该是下降,所以相反
(C) profit is irrelevant
(D) generic versions of drug 也还是 pscpt.drug, irrelevant
(E) according to premise 1, ALL drug prices are prohibited to rise, including foreign manufacturers.

36.    (32124-!-item-!-188;#058&006025),做错
[Background] – outsourcing
[Premise]
1.    Company V is experiencing a decline in profit
2.    Company V decides to outsourcing in order to increase its profit
[Conclusion] – V’s plan will achieve its goal

求support
[Prephrase] – company V找的到便宜的供应商

(A) 哪些不从外采购与本题无关
(B) irrelevant
(C) irrelevant
(D) 采取竞价不等于能找到低价的好供应商
(E) right answer

解题思路需揣摩
作者: TICKCOCK    时间: 2012-8-8 20:31
8月8日  【1-9】
1.    Originally, five senses are viewed as distinct yet complementary.
Premise: recent research has discovered that some people taste a banana and claim that they are tasting blue.
Conclusion: some people called synesthesiacs have senses that do not respect the usual boundaries.
Answer: C ( A)

2.    Caterpillar emerges from the egg on a tree branch immediately climbs upward until it finds a leaf bud to eat. Biologists though that this behavior displayed an innate tendency to move in the direction opposite to the pull of gravity.
However, a strong light source was placed at the bottom of a tree, caterpillar climbed downward.
Prephase, the biologist’s view is wrong.
Answer: C

3.    Background: for many years, per capita expenditure on prescription drugs in V rose.
Premise: the ministry prohibit drug manufactures from raising any of their products’ price.
Result: use of drug did not expand, but per capita expenditure rose.
Prephse: the per capita amount rose.
Answer: A

4.    rehistoric painting shows the deer had hump on its back. However, the fossil of deer shows no evidence of hump.
Answer: C

5.     Outsourcing is a method to run business. And V company plans to cast outsourcing those pars of its business that independent suppliers can provide at lower price.
Answer: E
作者: kathy0911    时间: 2012-8-9 14:54
1. 0:40Ppl have different senses and each sense can be stimulated by different things.
However, there are special cases that people with Sxx reported having a mixed respond to different senses.
Therefore they have sense that do not respect to the normal five senses


To weaken: something internal might be messed up?
 
(A) if they cannot understand the meaning of a word, their answers cannot be used to reflect their real feelings. ----correct
(B) there are other sense? ---support the conclusion
(C)there's a different pattern? -----irrelevant
(D)good to know but-----irrelevant
(E)good to know but-----irrelevant


Answer: A


2. 0:38
Butterfly climb up after emerged from the egg.
专家说:they climb to the opposite direction of the gravity
实验:they climb towards the light, although it is the same direction of the gravity


(A)if so, why they climb down when putting the light at the bottom? --------Wrong
(B)if they cannot see in the dark why climbing downwards?-----------incorrect
(C)this explain why they climb up and climb downwards when the light was at the bottom ------------correct answer
(D)really? then why they climb down in the experiment?--------------incorrect
(E)good to know but----irrelevant


Answer: C


3. 0:45
(A)although they produce other product, this does not means their price is increased?---irrelevant
(B)population increase? this should lower the per capital expenditure------Weaken
(C)----irrelevant
(D)gov encourage to buy more----correct
(E)stop marketing? this should decrease the sale of the drug and bring down the profit------Weaken

Answer: D
其实最后在纠结A和D不过现在想想如果是D那use of the drag should increase.... 但是还是不懂为啥A。。

4. 0:40
In the painting dear has large hump on the back, but fossil does not.
What is missing to link these two together?


(A)irrelevant
(B)irrelevant
(C) totally explain why large hump was not found in the fossil-------------correct!
(D)irrelevant
(E)irrelevant


Answer: C
5. 0:39
by outsourcing some of the service, V will gain profit.
To do so will: lower cost so increase profit.


(A)not guarantee a profit------incorrect
(B)irrelevant
(C)irrelevant
(D)irrelevant
(E) correct


Answer: E
作者: silviahelena    时间: 2012-8-10 15:29
报到...补昨天的作业....

1. Premise: The five senses have traditionally been viewed as
distinct yet complementary; However, recent
research has discovered that some people taste a
banana and claim that they are tasting blue, or see a
color and say that it has a specific smell
   Conclusion: synesthesiacs have senses
that do not respect the usual boundaries between the
five recognized senses.

Weaken: If this doesn't really mean synesthesiacs's senses overlap

Answer is A

33 Premise: C has an innate tendency to move in the direction opposite to the pull of gravity. When a strong light source was placed at the bottom of a tree, and caterpillars, after hatching, climbed downward.

Best Support: C is more sensitive to light

Answer is C

34: Premise: the per capita expenditure of prescription medicine is growing at 15% per year. The government has prohibited drug manufacturers from raising any of their products' prices. The use of p... drugs has not increased since then.

Conclusion: the per capita expenditure still raised

Explanation: Either the population decreased, or the total expense increased.

Answer is A

这题我其实一开始错了,直到我老公跟我解释说A的意思其实是政府对已有处方药限价,并不表示新的处方药也要限价. A 符合逻辑链中后面的一种情况。

35. Premise:the last giant deer lived in Ireland about 16,000 years ago. cave paintings in France depict this animal as having a large hump on its back. However, the fossils does not show the lump


Explanation: the painting must be before the last giant deer lived. And we do not know whether the painting is wrong. it could be that in certain period before 16,000, the giant deer did not have the large lump. but the fossils must be showing the real condition. it could also be the lump didn't become part of the fossil.

Answer is C

36. Background: Outsourcing is obtaining from an independent supplier a product or service that a company has previously provided for itself. Company V's profit has declined.
Premise: Some suppliers can provide at lower cost than V.
Conclusion: By outsourcing certain part to the vendor, V can achieve a higher profit

Support: this outsourcing can lower other cost or at least won't generate other cost

Answer is E.
作者: wanggang0411    时间: 2012-8-15 15:13
1 time 40
Premise: Each sense is incapable of stimulating the other senses.
Premise: recent research found that someone tastes a banana and claims it taste blue.
Conclusion: Have senses that do not respect the usual boundaries between the five recognized senses.
Inference: weakening the research found.

2 time 1:00
Premise: caterpillar climbs upwards after emerged from eggs.
Premise: Scientist explains that caterpillar climbs on the opposite direction of gravity.
Premise: In recent experiment, caterpillar climbs downward the tree which has a light source putted on the bottom.
Inference: Caterpillar climbs towards the light.
C

3 time 42
Premise: The expenditure of prescription drugs per capita rose 40 percent last year.
Premise: The price of prescription drugs doesn’t increase.
Premise: The expenditure of prescription of drugs per capita keeps rising.
Inference: More prescription drugs have been taken per capita.
There is no correct answer!!!
4 time: 42
Premise: deer has hump in the painting.
Premise: deer doesn’t have hump in its fossil.
Conclusion: There is no reason to conclude that the cave painting of deer is inaccurate, since__
Inference: Hump doesn’t consist of bones.
C: Humps are not composed of bones.
5 time: 42
Premise: What is outsourcing.
Premise: V experienced a decline in its profit.
Conclude: V will outsource those parts to independent suppliers which can provide at lower cost to increase its profit.
Inference: Yes, the cost has been reduced.
Both D and E are quite attractive, but since D mentioned bids which related to the lower cost indirectly, so I think D is the correct answer.
作者: wanggang0411    时间: 2012-8-15 15:32
3. OK, I missed a key point that use of prescription drugs did not expand.
So the plausible explanation will be the new medicine.
5. OK, more attention paid by manager, more profit will be earned by company.
But I still emphasize that this answer is also an indirect answer.
作者: chengzaaaa    时间: 2012-8-26 09:28
9.
1)36”weaken
2)BI: 5 senses are distinct yet complementary. Own stimuli that can’t stimulate others.
P: people taste a banana and claim that they taste blue, or see a color and say that it has smell.
C: these people’s senses don’t respect the usual boundaries between the five senses.
3)have inability to tell the five senses apart
4)E A
5)A. nothing to do with words
B. Range. Discussing these five senses, not others
C. not weakening
D.Range. and culture is irrelevant
E. Correct.

33.
1)28”couclusion
2)caterpillar, from egg, climb upward till a leaf to eat.
Thought: innate, opposite the pull of gravity
Recent: light at the bottom, after hatching, climb down
3)go with light
C

34.
1)29”explain
2)BI: per cap exp on pres drugs rise greatly annually.
P: not allow manufactures to raise price
C: uses did not expand, per cap expenditure still goes up greatly
3)doctors are prescribing more and more drugs.
A

35.
1)19”complete
2)BI: giant deer now extinct
P: painting: has a hump in the back. Fossils: not showing a hump
C: not that painting is inaccurate
3)hump made of fat that don’t become fossil
C

36.
1)22”support
2)BI: outsoure. V declined in profits
P: outsource the parts that independent supplier can provide at lower cost
C: boost its profits
3)no more transportation or other fees
E


谢谢lz!!!!
作者: ElenW    时间: 2012-10-7 10:01
精练(40'8)
Bg:The five sense have traditionally been viesed as distinct yet complementary.Each sense have its own range of stimul are able to stimulate the other sense.
pre:some people taste a banana and claim that they are tasting blue, or see a color and say that it has a specific smell.
con:Synesthesiacs people can not respect the usual boundaries between the five recognized sense.
(weaken)
Pre:The argument is not correct.This kind of people may has sth. woring in their mind.......
Ans:A
B-there is no issue
C-support
D-irrelevant
E-irrelevant
33(26'9)
Bg:When a caterpillar emerges from the egg on a tree branch, it immediately climbs upward until it finds a leaf bud to eat.
pre:this behavior displayed an innate tendency to move in the direction opposite to the pull of gravity.
con:a strong light source was placed at the bottom of a tree, and caterpillars, after hatching, climbed downward.
(strengthen)
Pre:to discuss that it is none of the gravity.
Ans:C
作者: srafcatt    时间: 2012-10-8 09:04
精炼:39s weaken
BG:senses--distinct and each own stimuli cannot stimulating others
premise:someone say sth strange
conclusion:their senses don't respect the common boundaries.

this is their occasional cases,not the general ones.

choice:E-------------------------------A
A irrelevant
B strenghten
C strenghten
D irrelevant
E correct
本题启示:做的太匆忙,没看好答案就选,首先是weaken,然后E项说的是他们只有少数时间能脱离那种幻觉状态。因此是无法weaken的。
A 项说的是听觉受损,理解东西存在问题。恰好就weaken了argument。

逻辑链:
1.35s assumption
BG:caterpillar tends to climb upwards after hatching.
situation:caterpillars climb downward towards the light source.

light source is the real factor to lead the caterpillar's climbing direction.
choice:C
A that explanation is opposite the former observations.
B not exactly and irrelevant
C correct
D not necessarily
E irrelevant

2.65s paradox
premise:ministry forbid the raises of products' price
but:though the use of drug does not expand,the percentage increases.

explanation:the depression happens again
choice:C
A new medication can be observed if their quantity is vast.
B per capita means the population is irrelevant with this argment
C correct
D conflict with the argument
E weaken the argument

3.26s fill
premise:****
conclusion:the painter paint hump but it cannot be suggested by its fossil.

the premise can be there exist no bones in hump,so it decayed.
choice;C
A conflict with the argument
B irrelevant
C correct
D irrelevant
E not convincing

4.30s support
premise:vernon Inc. outsourcing
conclusion:profits increase

this corporation focus on its main career and earns profits more effectively.
choice:E
A irrelevant
B irrelevant
C irrelevant
D irrelevant
E correct
作者: srafcatt    时间: 2012-10-8 09:15
纠错:第34题我选C了,开始特别委屈,不选C 选谁呀。
 A 对是因为既然老药限价,那么药品开发商开发新药,新药可以随意涨价,因此政府失算了。
C 错是因为问题的中心问的是为什么政府的计划失算了?不是问为什么医药开发商的利润又涨了?政府很高兴医药开发商的利润增长,但是不能危害百姓利益。不是不愿意看见其利润增长。搞懂这个之后,C 其实是一个答非所问的无关选项。
作者: hanhan1991    时间: 2012-11-21 12:02
1.weaken
1)time:35'/1''17'
2)logical lines:
Background:The five senses of people are believed to be  distinct from one the another.
Premise:Some people taste a banana and claim they are testing blue or see a color and say that it has a specific smell.
Conclusion: The senses of these people are overlapped.
3)Predication:
If these people can not distinguish the real sense they feel, in other words they may feel the sense of tasty as blue, the conclusion can be weakened.
4)Answer:A
A.correct. the phynomenon can be explained by their inability to use and understand words rather than the overlapped sense.
B.other sense is irrelevant.
C.how the sense is overlap is irrelevant.
D.how the phemonon is described is irrelevant.
E.how to rid of the synesthetic experience is irrelevant.

2.must be true
1)time:39'/57'
logical lines:
Fact1:After hatching, a caterpillar climbs upward -> this is explained by biologist that the animal's innate tendency to move in the direction opposite to the pull of gravity.
Fact2:When a strong light source is put at the bottom of the tree, the caterpillar climb downward.
3)Prediction:
The caterpillar's moving direction is affected by the source of light rather than the pull of gravity.
4)Answer:C
A.it opposites to the fact2.
B.unable to see in the dark does not mentioned.
C.it is the same as the prediction.
D.it can not be inferred.
E.it can not be inferred.

3.explain
1)time:39'/1''14'
2) :
Backgrounder capital expenditure on prescreption drugs in V rose at a dramatic rate.
Fact: The minister prohibit drug manufacturer from rasing any of their products's price.
Paradoxer capital expenditure on prescription drugs contine to increase by a substantial percentage.
3)Prediction:
If drug manufacturers invent many news drugs of high price and these drugs are prescribed by doctors , the paradox can be explained.
4)Answewr:A
A. it is similar to the predication.
B.The population is irrelevant.
C.the profits level of manufacturer is irrelevant.
D.it aggravate the paradox.
E.it is irrelevant.

4.explain
1)time:27'/52'
2)
Factaintings of gaint deer had a large hump on its back.
Paradox:Fossils of the animal do not contain such hump though the painting are acurate.
3)Prediction:
Gain deer has hump but it fossil does not have, so the hump can not exsit in the form in the fossil.
4)Answer:C
A.this aggravate the paradox.
B.irrelevant.
C.the same as prediction
D.irrelevant.
E.this aggravate the paradox.

5.strengthen
1)time:33'/1''33'
2)logical lines:
Objective:To increase its profits
Plan:Outsourcing its business to independent suppliers with lower costs.
3)Prediction:
If outsourcing can make the company gain at least as much revenue as it does the business by itself, the plan will succeed.
4)Answer:E
A.irrelevant,since we only concern about the outsourcing part.
B.irrelevant.
C.it weakens the argument.
D.how to select is irrelevant.
E.it say the advantage of outsourcing thus strengthen the argument.
作者: zxppx    时间: 2013-1-12 10:26
9. The five senses have traditionally been viewed as
distinct yet complementary. Each sense is thought to
have its own range of stimuli that are incapable of
stimulating the other senses. However, recent
research has discovered that some people taste a
banana and claim that they are tasting blue, or see a
color and say that it has a specific smell. This shows
that such people, called synesthesiacs, have senses
that do not respect the usual boundaries between the
five recognized senses.

Please note that the irrational behavior is related to the ability to use and understand words. The reason why I do wrong is that I overlook the meaning of the sentence, just thinking that if a person taste something, he or she cannot talk at the same time.
Which one of the following statements, if true, most
seriously weakens the argument?

(A) Synesthesiacs demonstrate a general,
systematic impairment in their ability to use
and understand words.

Even though we can see the impairment in their ability to use and understand words, the answer choice does not provide explanations to weaken the argument, considering their ability to feel smell while see a color.

The argument needs to demonstrate that the phenomenon which happens to synesthesiacs cannot weaken the common belief that the five senses are distinct yet complementary. So if synesthesiacs have an impairment in their ability to use and understand words, the facts that they taste a banana and claim that they are tasting blue, and that they see a color and say that it has a specific smell, may be not the reflection that they actually feel, but the reflection that they inaccurately response. So in this case, it does undermine the conclusion that the senses of synesthesiacs do overlap.
(B) Recent evidence strongly suggests that there
are other senses besides sight, touch, smell,
hearing, and taste.

The answer choice cannot weaken the argument, because the other senses besides the five senses are irrelevant.
(C) The particular ways in which sensory
experiences overlap in synesthesiacs follow a
definite pattern.

If the answer is true, it helps to demonstrate although synesthesiacs can feel other senses while contact one object, these senses are still distinct, considering the overlap of senses. BA

Actually, this choice strengthens the argument by pointing out that the particular ways follow a definite pattern.
(D) The synesthetic phenomenon has been
described in the legends of various cultures.

Even though the synesthetic phenomenon has been viewed as legends of various cultures, it still can happen so as to strengthen the argument, rather than weaken.
(E) Synesthesiacs can be temporarily rid of their
synesthetic experiences by the use of drugs.

There is no relationship with the way to temporarily get rid of the synesthetic experiences.





33. (31934-!-item-!-188;#058&005658)
When a caterpillar emerges from the egg on a tree branch, it immediately climbs upward until it finds a leaf bud to eat. Biologists thought that this behavior displayed an innate tendency to move in the direction opposite to the pull of gravity. In a recent experiment, a strong light source was placed at the bottom of a tree, and caterpillars, after hatching, climbed downward.
Which of the following hypotheses is best supported by the statements given?
(A) Caterpillars have an innate tendency to move in the direction of gravity.

Although it is one of the hypotheses referred in the argument, it is not the one that can be best supported by the statements given.
(B) Newly hatched caterpillars are unable to see in the dark.

Nothing provided can demonstrate the choice.
(C) Newly hatched caterpillars move towards the strongest light source in the environment.

Firstly, biologists assume that the gravity can influence. Then, after the strong light experiment, they find that the answer choice can be more accurately demonstrated. BA
(D) Newly hatched caterpillars move toward the leaf bud nearest to them.

The same to B.
(E) The eyes of newly hatched caterpillars become less sensitive to light over time.

It has the opposite conclusion with the experiment referred.



34. (31982-!-item-!-188;#058&005681)
For several years, per capita expenditure on prescription drugs in Voronia rose by fifteen percent or more annually. In order to curb these dramatic increases, the ministry of health prohibited drug manufacturers from raising any of their products' prices. Even though use of prescription drugs did not expand after this price freeze, per capita expenditure for prescription drugs continued to increase by a substantial percentage each year.
Which of the following, if true, most helps to explain why the ministry's action did not achieve its goal?
(A) After price increases were prohibited, drug manufacturers concentrated on producing new medications to replace existing products.

If the answer choice is true, it can help to explain the reason why the ministry’s action did not achieve its goal. Because when drug manufactures cannot increase products’ prices, they will switch from old products to new medications, which can be targeted at higher prices to make more profits. BA
(B) The population of Voronia rose steadily throughout the period.

Although more people there are in Voronia, more total expenditures will the country have. But the argument talks about per capita expenditure, rather than total expenditure.
(C) Improvements in manufacturing processes enabled drug manufacturers to maintain high profit levels on drugs despite the price freeze.

Even though the answer choice is so amazing that many people will choose, it does not catch the nucleus of the argument. Because the argument talks about why per capita expenditure continued to increase while the government has prohibited drug manufactures from arising any of their products’ prices, the answer choice talks about the reason why drug manufactures can still make a high profits during this situation, which has nothing to do with the argument.
(D) In addition to imposing a price freeze, the government encouraged doctors to prescribe generic versions of common drugs instead of the more expensive brand-name versions.

The choice strengthens the contradiction.
(E) After price increases were prohibited, some foreign manufacturers of expensive drugs ceased marketing them in Voronia.

The same to D.

35. (32076-!-item-!-188;#058&006018)
Which of the following most logically completes the argument?
The last members of a now-extinct species of a European wild deer called the giant deer lived in Ireland about 16,000 years ago. Prehistoric cave paintings in France depict this animal as having a large hump on its back. Fossils of this animal, however, do not show any hump. Nevertheless, there is no reason to conclude that the cave paintings are therefore inaccurate in this regard, since __________.
(A) some prehistoric cave paintings in France also depict other animals as having a hump

There is no relationship with other animals depicted in the prehistoric cave paintings.
(B) fossils of the giant deer are much more common in Ireland than in France

The comparison does not help to demonstrate the argument.
(C) animal humps are composed of fatty tissue, which does not fossilize

If animal humps cannot fossilize, there will not be any fossils of humps, thus helping complete the argument. BA
(D) the cave paintings of the giant deer were painted well before 16,000 years ago

It has no relationship with the time of these cave paintings.
(E) only one currently existing species of deer has any anatomical feature that even remotely resembles a hump

Actually, this answer choice weakens the argument.



36. (32124-!-item-!-188;#058&006025)
Outsourcing is the practice of obtaining from an independent supplier a product or service that a company has previously provided for itself. Vernon, Inc., a small manufacturing company that has in recent years experienced a decline in its profits, plans to boost its profits by outsourcing those parts of its business that independent suppliers can provide at lower cost than Vernon can itself.
Which of the following, if true, most strongly supports the prediction that Vernon's plan will achieve its goal?
(A) Among the parts of its business that Vernon does not plan to outsource are some that require standards of accuracy too high for most independent suppliers to provide at lower cost than Vernon can.

There is no relationship with the parts that Vernon does not plan to outsource.
(B) Vernon itself acts as an independent supplier of specialized hardware items to certain manufacturers that formerly made those items themselves.

It cannot strengthen the argument, even though we know what Vernon acts to certain manufactures.
(C) Relatively few manufacturers that start as independent suppliers have been able to expand their business and become direct competitors of the companies they once supplied.

The answer choice seems to be valid at first glance, while a more thorough analysis reveals that the choice is questionable. Because the argument refers to the goal that Vernon needs to boost its profits, but the choice talks about the bad effect of outsourcing can be eliminated, the answer cannot support the argument.
(D) Vernon plans to select the independent suppliers it will use on the basis of submitted bids.

The answer still cannot support the argument. What if the bidder cannot produce the parts at a lower cost than Vernon does itself?
(E) Attending to certain tasks that Vernon performs relatively inefficiently has taken up much of the time and effort of top managers whose time would have been better spent attending to Vernon's core business.


In order to make more profits, the managers will spend more time in the core business, thus strengthening the argument. BA

作者: CD用户825193    时间: 2013-1-13 17:18
标题: Daily CR-9_2013-01-13
BTW, 赶脚楼上滴好厉害的说。。。


9. The five senses have traditionally been viewed as

distinct yet complementary. Each sense is thought to
have its own range of stimuli that are incapable of
stimulating the other senses. However, recent
research has discovered that some people taste a
banana and claim that they are tasting blue, or see a
color and say that it has a specific smell. This shows
that such people, called synesthesiacs, have senses
that do not respect the usual boundaries between the
five recognized senses.
Which one of the following statements, if true, most
seriously weakens the argument?
   37" (Weak)
P: Traditional view of the five senses is that they are separated form each other.
P: Some people said they see blue or other color when they tast banana.
C: Those people have special connection between diffrent senses.
Pre: It's just imagination, nothing with neuro system.

Answers:
(A) Synesthesiacs demonstrate a general,
systematic impairment in their ability to use
and understand words.
   Irrelevant: it's not about ability to use words.
CORRECT: impairment in the ability to use and understand words -> not a reflection of the real senses they're experience -> still follow the usual boundaries.
(B) Recent evidence strongly suggests that there
are other senses besides sight, touch, smell,
hearing, and taste.
   Irrelevant: other senses is not helpful to the subject (among the five senses)

(C) The particular ways in which sensory
experiences overlap in synesthesiacs follow a
definite pattern.
   Not directly weaken.

(D) The synesthetic phenomenon has been
described in the legends of various cultures.
   Kind of support.

(E) Synesthesiacs can be temporarily rid of their
synesthetic experiences by the use of drugs.
   CORRECT: Those ppl had that experience because of other reason.
"be rid of" = "get rid of"
Totally midunderstood the meaning... this one kinda support the argument.

33. When a caterpillar emerges from the egg on a tree branch, it immediately climbs upward until it finds a leaf bud to eat.  Biologists thought that this behavior displayed an innate tendency to move in the direction opposite to the pull of gravity.  In a recent experiment, a strong light source was placed at the bottom of a tree, and caterpillars, after hatching, climbed downward.
Which of the following hypotheses is best supported by the statements given?

26" (Weak / Support)
P: Caterpillar climbs upwards on a tree when it reaches a leave to eat. Scientists thought caterpillar's nature was to against the gravity.
P: In a recent experiment, caterpillar put under the tree went down to the root.
C:
Pre: The caterpillar goes to where the food is.

Answers:(A) Caterpillars have an innate tendency to move in the direction of gravity.
(B) Newly hatched caterpillars are unable to see in the dark.
(C) Newly hatched caterpillars move towards the strongest light source in the environment. CORRECT
(D) Newly hatched caterpillars move toward the leaf bud nearest to them.
(E) The eyes of newly hatched caterpillars become less sensitive to light over time.



34. For several years, per capita expenditure on prescription drugs in Voronia rose by fifteen percent or more annually.  In order to curb these dramatic increases, the ministry of health prohibited drug manufacturers from raising any of their products' prices.  Even though use of prescription drugs did not expand after this price freeze, per capita expenditure for prescription drugs continued to increase by a substantial percentage each year.
Which of the following, if true, most helps to explain why the ministry's action did not achieve its goal?


31" (Support)
P: Per capita expenditure for prescription drugs rose annually.
P: Ministry prohibits the drugs manufacturers from increasing the prices,
C: but though the use of prescription drugs didn't expand, the per capita expenditure continues increasing each year.
Pre: less ppl in the area

Answers:
(A) After price increases were prohibited, drug manufacturers concentrated on producing new medications to replace existing products.有关这个选项,纠结了好久才想通:
之前觉得,政府不是限制涨价了么,那么不管新药、旧药,都不应该涨价才对啊。。。
最后想通:特么的药商,不让要涨价,我就生产更贵的新药啊,旧药不涨价了,但是新药虽然与旧药功效一样,但是更贵了啊,其实也就是治疗同样疾病的东西,还是涨价了。。。奸商啊奸商!!!
(B) The population of Voronia rose steadily throughout the period.
(C) Improvements in manufacturing processes enabled drug manufacturers to maintain high profit levels on drugs despite the price freeze.
(D) In addition to imposing a price freeze, the government encouraged doctors to prescribe generic versions of common drugs instead of the more expensive brand-name versions.
(E) After price increases were prohibited, some foreign manufacturers of expensive drugs ceased marketing them in Voronia.
CORRECT? (Not vr sure, but others not make sense.)


35.Which of the following most logically completes the argument?


The last members of a now-extinct species of a European wild deer called the giant deer lived in Ireland about 16,000 years ago.  rehistoric cave paintings in France depict this animal as having a large hump on its back.  Fossils of this animal, however, do not show any hump.  Nevertheless, there is no reason to conclude that the cave paintings are therefore inaccurate in this regard, since __________.

32" (Support)
P: A now-extinct deer was painted having hump on its back.
P: Fossils of the deer doesn't have hump.
Pre: Missing Piece: Hump doesn't last as long as to become fossil.
C: The painting correctly describes the deer.

Answers:
(A) some prehistoric cave paintings in France also depict other animals as having a hump
(B) fossils of the giant deer are much more common in Ireland than in France
(C) animal humps are composed of fatty tissue, which does not fossilize
(D) the cave paintings of the giant deer were painted well before 16,000 years ago
(E) only one currently existing species of deer has any anatomical feature that even remotely resembles a hump



36.Outsourcing is the practice of obtaining from an independent supplier a product or service that a company has previously provided for itself.  Vernon, Inc., a small manufacturing company that has in recent years experienced a decline in its profits, plans to boost its profits by outsourcing those parts of its business that independent suppliers can provide at lower cost than Vernon can itself.
Which of the following, if true, most strongly supports the prediction that Vernon's plan will achieve its goal?

24" (Support)
P: Company X is losing profits.
P: X tries to outsource the parts to other company which has lower cost
C: to increase X's profits.
Pre: Outsourcing doesn't cause extra fee like management fee or quality issue which might lower the market of its products.

Answers:

(A) Among the parts of its business that Vernon does not plan to outsource are some that require standards of accuracy too high for most independent suppliers to provide at lower cost than Vernon can.
(B) Vernon itself acts as an independent supplier of specialized hardware items to certain manufacturers that formerly made those items themselves.
(C) Relatively few manufacturers that start as independent suppliers have been able to expand their business and become direct competitors of the companies they once supplied.
(D) Vernon plans to select the independent suppliers it will use on the basis of submitted bids.
(E) Attending to certain tasks that Vernon performs relatively inefficiently has taken up much of the time and effort of top managers whose time would have been better spent attending to Vernon's core business.
作者: pennyz    时间: 2013-2-11 23:12
1:57s
e
background:the five sense is seperate to each other
premise:there are people who had syne
conclusion:have sense do not respect usual boundary
rephrase:it is actually a disease
(A) Synesthesiacs demonstrate a general, systematic impairment in their ability to use
and understand words.----define wrong,irrelevant
(B) Recent evidence strongly suggests that there are other senses besides sight, touch, smell, hearing, and taste.------strengthen
(C) The particular ways in which sensory experiences overlap in synesthesiacs follow a
definite pattern.-------strengthen,because it form the pattern
(D) The synesthetic phenomenon has been described in the legends of various cultures.---irrelevant
(E) Synesthesiacs can be temporarily rid of their synesthetic experiences by the use of drugs.--------it is temporary
作者: pennyz    时间: 2013-2-11 23:31
1:10s
c
phenomenon
if the caterpillar were set on the branch,it climb up to eat the first bud
if the strong light source  were set on the bottom,caterpillar climb downward
3:08
a
aim:to decrease the increase of prescribtion drugs
method:prohibit d m f raising price
phenomenon:no not expand
per unit price increase
reason:?

1:05s
c
a:the other animal has hump is irrelevant/if we have mistaken,too far away
birrelevant
c correct
d time ,irrelevant
e current exist deer ,irrelevant
2:07
e
aim:increase profit
methodutsource the part that cost more for the company
rephrase:it is high cost that cause the decline of the profit
作者: 纽约我爱你    时间: 2013-3-30 21:06
1~28s
Premise1. the five stimulates are not related
2.some people can combine them
Conclusion: these people have other senses other than the usual boundaries among the five
Answer: C 1’13’’
正解:A(我发现看不清出题真的是硬伤,如果读懂了真的还好)
2~38s
Premise: 1.an insect will climb a tree until it finds a leaf and eats it when it is born.
2.when there’s a light source at the bottom of a tree, the insect will climb down
Conclusion: the previous thought that it moves against the gravity is wrong
Supported: it moves towards the light
Answer: C 22’’
3~39s……totally理解错题意了!!!!!
Premise: the government raises the price of a drugthe government forbid the rising of the price of drugs
Conclusion: the use of the drug per person increases.(although the use of the drugs doesn’t change, the spend on drug per person increases)
Explain: the illegal market of it grows(doctors make other prescriptions to make more profits)
Answer: E 不算时间了吧……大惨败……
什么……竟然选的答案是错的……我屮艸芔茻……
正解是A。难道是说新药的成本贵一些吗……
PS.cease是停止的意思啊姑娘,背背单词再来选吧>_<
4~40s
Premise: 1.the picture of an animal is found in a cave
2.the back of the animal has a horn
3.we have never found such a fossil
Conclusion: we can’t say that the picture is inaccurate
Explain: other strange animals that are pictured on the cave have been found
Answer: C 34’’
5~30s
Premise: a company suffers from a decline in profit
Conclusion: the company prepare to depend something they need on another company which can make it with less money
Support: the company can use the money to make more profitable produces
Answer: E 1’17’’

作者: okplokpl0714    时间: 2013-4-2 21:49
精练:
Five senses distinct yet complementary.
Each sense have own range of stimuli incapable of stimulating the other senses.
BUT!!recent research:taste banana tasting blue;see a color feel specific smell.
This show symesthesiacs have sense not boundaries between senses.
削弱猜测:他因,可能在测试之前没有排除一些因素。
答案看的好累啊。。不懂。。排除一下猜个C吧。。。
果然选错。。好好看分析

逻辑链:
33.
caterpillar emerges-->climbs upward until find leaf bud to eat.
B:this behavior displayed an innate tendency to move in the direction opposite to pull of gravity.
Recent experiment,strong light placed at the bottom--->caterpillars downward.
加强猜测:猜不出来。。。。囧
Answer:E
答案是C

34.
For several years,per capita predrugs in V rose by 15% or more annually.
ministry of health prohibited drug manufacturers from rising any of products' prices to curb increases
though use of predrugs did not expand after prise freeze,per capita expenditure continued to increase
解释猜测:供小于求
Answer:A

35.
E giant deer in I about 16000years ago.
cave paintings in F show giant deer have large hump on back
BUT!!Fossils of giant deer not show any hump.
BUT!!there is no reason to conclude that the cave paintings inaccurate ,since---
解释猜测:化石可能搞错了,有可能是另一个品种的鹿
Answer:C

36.
Outsourcing is ....
VI recent decline in profits,plan to boost by outsourcing suppliers can provide at lower cost than V
加强猜测:V自己做会很费钱
Answer:A
答案是E

作者: Feelalive    时间: 2013-7-9 14:44
留爪印~~~~~~~~~~~~~~~~~~~~~
作者: Mint静默    时间: 2013-8-10 10:38
Bible的题目还真抽象=。=
1) 计时:37s
2) 逻辑链
Background Information:
The five senses have traditionally been viewed as distinct yet complementary. Each sense is thought to have its own range of stimuli that are incapable of stimulating the other senses.
Premise:
some people taste a banana and claim that they are tasting blue, or see a color and say that it has a specific smell.
Conclusion:
such people, called synesthesiacs, have senses that do not respect the usual boundaries between the five recognized senses.
3) 推测:
选项可能直接反驳结论,证明这些人respect the usual boundaries between the five recognized senses. 或者 说这些人存在的问题其实在另一个方面(它因)
4) 选项分析:
(A) Synesthesiacs demonstrate a general, systematic impairment in their ability to use and understand words. 它因,他们不是分不清senses, 而是词不达意
(B) Recent evidence strongly suggests that there are other senses besides sight, touch, smell, hearing, and taste. 脱离样本范围,无关
(C) The particular ways in which sensory experiences overlap in synesthesiacs follow a definite pattern. 与pattern无关
(D) The synesthetic phenomenon has been described in the legends of various cultures. Origins of synesthetic phenomenon无关
(E) Synesthesiacs can be temporarily rid of their synesthetic experiences by the use of drugs. How to solve synesthetic 无关

1) 计时:24s
2)
逻辑链
Premise1:
When a caterpillar emerges from the egg on a tree branch, it immediately climbs upward until it finds a leaf bud to eat.
Premise2:
A strong light source was placed at the bottom of a tree, and caterpillars, after hatching, climbed downward.
3) 推测:
New-born caterpillar was attracted by the strongestlight near it
So C is correct

1)
计时:27
s
2)
逻辑链
Background Information:
For severalyears, per capita expenditure on prescription drugs in Voronia rose by fifteenpercent or more annually.
Premise:
use of prescription drugs did notexpandafter this price freeze
Conclusion X:
per capita expenditure for prescription drugs continued to increase by a substantialpercentage each year.
3) 推测:
use of prescription drugs did notexpand but doctor prescribed more drugs(other reasons)
B is correct

1) 计时:29s
2)
逻辑链
Premise:
Prehistoric cave paintings in France depict this animal as having a large hump on its back. Fossils of this animal, however, do not show any hump.
Conclusion X:
There is no reason to conclude that the cave paintings are therefore inaccurate in this regard,C is correct

1) 计时:18s
2)
逻辑链
Background Information:
Outsourcing is the practice of obtaining from an independent supplier a product or service that a company has previously provided for itself.
Plan:
outsourcing those parts of its business that independent suppliers can provide at lower cost than Vernon can itself.
Goal:
boost its profits
3) 推测(Support):
排除outsource可能带来的某一方面成本增加的可能性
E is correct


作者: Elisha728    时间: 2013-8-29 06:52
6'24''
CCACE
作者: Elisha728    时间: 2013-8-29 06:54
balapupu 发表于 2011-11-27 22:08
Question #4. June 2001 LSAT, Section #1, #12. The correct answer choice is (A)The conclusion is in t ...

第一题错了……。
作者: lyrsilvia    时间: 2013-9-16 19:35
1. 32'
BG: The five senses have traditionally been viewed as distinct yet complementary. Each sense is thought to
have its own range of stimuli that are incapable of stimulating the other senses.
P: recent research has discovered that some people taste a banana and claim that they are tasting blue, or see a color and say that it has a specific smell.
C: such people, called synesthesiacs, have senses that do not respect the usual boundaries between the five recognized senses.
Weaken: claim taste is not real taste
答案:B
(A) Synesthesiacs demonstrate a general, systematic impairment in their ability to use and understand words.----------what synesthesiacs demonstrate is not relevant, ability to use and understand words is not discussed in the question. CORRECT
(B) Recent evidence strongly suggests that there are other senses besides sight, touch, smell, hearing, and taste.-----------means that the specific feeling is not cross boundaries, but another unknown sense
(C) The particular ways in which sensory experiences overlap in synesthesiacs follow a definite pattern.-------------whether synesthesiacs follow a definite pattern is not discussed, and not useful to justify the boundaries
(D) The synesthetic phenomenon has been described in the legends of various cultures.------------strengthen the statement about the emerge of synethestic
(E) Synesthesiacs can be temporarily rid of their synesthetic experiences by the use of drugs.----------whether we will rid of synethetic experiences does not affect the synesthesiacs itself
===============================================================
怎么精炼总错...
方向: sth. that would suggest their sense do respect the usual boundaries
A项意思是even they express, they did not express precisely. what they express cannot be taken seriously. even they said they tasting blue or see a color, it does not mean they actually experience it.
SUPPORT
===============================================================
2. 17'
P:  recent experiment, a strong light source was placed at the bottom of a tree
C: caterpillars, after hatching, climbed downward.--> means not move in the direction opposite to the pull of gravity.
Support: light source cause the caterpillars to move downward
答案:
(A) Caterpillars have an innate tendency to move in the direction of gravity.------------weaken C
(B) Newly hatched caterpillars are unable to see in the dark.--------------whether they are able to see is not affect their moving
(C) Newly hatched caterpillars move towards the strongest light source in the environment.----------CORRECT, means the lights matter
(D) Newly hatched caterpillars move toward the leaf bud nearest to them.------------do not know whether the leaf is upward or downward
(E) The eyes of newly hatched caterpillars become less sensitive to light over time.-----------less sensitive means not the lights cause the downward moving. Weaken.

3. 28’
P:  to curb these dramatic increases, the ministry of health prohibited drug manufacturers from raising any of their products' prices
C: per capita expenditure for prescription drugs continued to increase by a substantial percentage each year
Explain: prohibited raise prices is not effective to curb the per capita expenditure-->other factors affect the per capita
答案:A
(A) After price increases were prohibited, drug manufacturers concentrated on producing new medications to replace existing products.---------------new medications is another factor that affect the per capita consumption. CORRECT
(B) The population of Voronia rose steadily throughout the period.--------------if more population, the per capita consumption should be lower. because (whole consumption)/population= per capita consumption
(C) Improvements in manufacturing processes enabled drug manufacturers to maintain high profit levels on drugs despite the price freeze.---------------profit is irrelevant to consumption
(D) In addition to imposing a price freeze, the government encouraged doctors to prescribe generic versions of common drugs instead of the more expensive brand-name versions.------------this means per capita consumption should decline but not increase
(E) After price increases were prohibited, some foreign manufacturers of expensive drugs ceased marketing them in Voronia.---------------------expensive ceased means people spend less.
个人觉得A中是不是隐藏了new medication是不会be prohibited from raising price的条件

4. 15’
P:  Fossils of this animal, however, do not show any hump.
C: no reason to conclude that the cave paintings are therefore inaccurate in this regard
Explain: fossils cannot prove the reality
答案:C
(A) some prehistoric cave paintings in France also depict other animals as having a hump--------whether other animals have hump or not is sufficient to know whether deer have hump
(B) fossils of the giant deer are much more common in Ireland than in France----------where the fossils are more common is insufficient to know whether deer have hump
(C) animal humps are composed of fatty tissue, which does not fossilize-----------CORRECT. means fossils are not reflecting the truth
(D) the cave paintings of the giant deer were painted well before 16,000 years ago------------when were the fossils painted is insufficient to know whether deer have hump
(E) only one currently existing species of deer has any anatomical feature that even remotely resembles a hump------------------------currently existing species cannot guarantee the past species.

5. 28'
P: outsourcing those parts of its business that independent suppliers can provide at lower cost than Vernon can itself.
C: V can boost its profits
Support: outsourcing can boost its profits, reduce its cost
答案:
(A) Among the parts of its business that Vernon does not plan to outsource are some that require standards of accuracy too high for most independent suppliers to provide at lower cost than Vernon can.-----------we just consider the outsourcing part not the other part
(B) Vernon itself acts as an independent supplier of specialized hardware items to certain manufacturers that formerly made those items themselves.------------what V acts does not related to its action, we will not know the outsourcing is sufficient or not
(C) Relatively few manufacturers that start as independent suppliers have been able to expand their business and become direct competitors of the companies they once supplied.------------illustrate the safe of outsourcing
(D) Vernon plans to select the independent suppliers it will use on the basis of submitted bids.----------how it select suppliers does not show outsourcing is effective or not
(E) Attending to certain tasks that Vernon performs relatively inefficiently has taken up much of the time and effort of top managers whose time would have been better spent attending to Vernon's core business.------------CORRECT. means top managers can be more effective to core business to make profit for V.
作者: meckyona    时间: 2013-9-29 22:31
9
26s
Some people have senses that don’t respect the usual boundaries between 5 senses

Weaken
A words are irrelevant
B other sights are irrelevant
C
D irrelevant
E irrelevant
一遍看下来觉得都是无关的orz 选C吧…
33
31s
B:a caterpillar emerges from an egg and climb up
C a behavior that opposite to the pull of gravity

A but without light it moves in the opposite way
B irrelevant
C true, right answer
D no, it can be anywhere
E irrelevant

34
35s
B the expenditure on p drug rose annually
P the ministry of health ban the manufactures from raising any prices
C the use didn’t expand but the expenditure continue to increase

A irrelevant
B more people, more use of drugs..,right
C ….这个不知道怎么分析
D doesn’t explain
E irrelevant

35
A can’t complete
B irrelevant
C right
D irrelevant
E no

36
A irrelevant
B irrelevant
C irrelevant
D
E
好奇怪= =选D吧

作者: meckyona    时间: 2013-9-29 22:37
meckyona 发表于 2013-9-29 22:31
9
26s
Some people have senses that don’t respect the usual boundaries between 5 senses

精练题的A,直接攻击word

per capita 是人均的意思, new drug没有价格限定= =

作者: 览物之情    时间: 2013-10-11 05:33

10月10号

1 27s  Background:The five senses have traditionally been viewed as distinct yet complementary. Each sense is thought to have its own range of stimuli that are incapable of stimulating the other senses.
Premise :However, recent research has discovered that some people taste a
banana and claim that they are tasting blue, or see a color and say that it has a specific smell.
Conclusion:This shows that such people, called synesthesiacs, have senses that do not respect the usual boundaries between the five recognized senses.

Which one of the following statements, if true, most
seriously weakens the argument?
因果推理,干扰因素P maybe those people just do not express their felling well.
A)        Synesthesiacs demonstrate a general, systematic impairment in their ability to use
and understand words.-----------correct
(B) Recent evidence strongly suggests that there are other senses besides sight, touch, smell, hearing, and taste.--------support
(C) The particular ways in which sensory experiences overlap in synesthesiacs follow a
definite pattern.--------support
(D) The synesthetic phenomenon has been described in the legends of various cultures.------support or some kind of irrelevant
(E) Synesthesiacs can be temporarily rid of their synesthetic experiences by the use of drugs.------------irrelevant.

2  24s
Background: When a caterpillar emerges from the egg on a tree branch, it immediately climbs upward until it finds a leaf bud to eat.  Biologists thought that this behavior displayed an innate tendency to move in the direction opposite to the pull of gravity.

Premise: In a recent experiment, a strong light source was placed at the bottom of a tree, and caterpillars, after hatching, climbed downward.

Which of the following hypotheses is best supported by the statements given?
问conclusion: maybe light is the cause that make C climb.


(A) Caterpillars have an innate tendency to move in the direction of gravity.
(B) Newly hatched caterpillars are unable to see in the dark.
(C) Newly hatched caterpillars move towards the strongest light source in the environment.----correct
(D) Newly hatched caterpillars move toward the leaf bud nearest to them.
(E) The eyes of newly hatched caterpillars become less sensitive to light over time.

3 25s
Background:For several years, per capita expenditure on prescription drugs in Voronia rose by fifteen percent or more annually.

Plan: In order to curb these dramatic increases, the ministry of health prohibited drug manufacturers from raising any of their products' prices.

Outcome: Even though use of prescription drugs did not expand after this price freeze, per capita expenditure for prescription drugs continued to increase by a substantial percentage each year.
Which of the following, if true, most helps to explain why the ministry's action did not achieve its goal?
Price and quantity unchanged then maybe the capital expenditure of other thing decrease?

(A) After price increases were prohibited, drug manufacturers concentrated on producing new medications to replace existing products.-------correct.很可能新药价格更高
(B) The population of Voronia rose steadily throughout the period.-------irrelevant
(C) Improvements in manufacturing processes enabled drug manufacturers to maintain high profit levels on drugs despite the price freeze.--------irrelevant
(D) In addition to imposing a price freeze, the government encouraged doctors to prescribe generic versions of common drugs instead of the more expensive brand-name versions.-----更矛盾了
(E) After price increases were prohibited, some foreign manufacturers of expensive drugs ceased marketing them in Voronia.-----irrelevant.

4 27s
Which of the following most logically completes the argument?


The last members of a now-extinct species of a European wild deer called the giant deer lived in Ireland about 16,000 years ago.   rehistoric cave paintings in France depict this animal as having a large hump on its back.  Fossils of this animal, however, do not show any hump.  Nevertheless, there is no reason to conclude that the cave paintings are therefore inaccurate in this regard, since __________.
相关因果。Fossils might lose some part. Or hump is easy to be destroyed.
(C) animal humps are composed of fatty tissue, which does not fossilize------correct

5 22s
Background: Outsourcing is the practice of obtaining from an independent supplier a product or service that a company has previously provided for itself.
Premise: Vernon, Inc., a small manufacturing company that has in recent years experienced a decline in its profits, plans to boost its profits by outsourcing those parts of its business that independent suppliers can provide at lower cost than Vernon can itself.

Which of the following, if true, most strongly supports the
conclusion: that Vernon's plan will achieve its goal?
P : no side effect有效性,可行性。

(A) Among the parts of its business that Vernon does not plan to outsource are some that require standards of accuracy too high for most independent suppliers to provide at lower cost than Vernon can.-------irrelevant
(B) Vernon itself acts as an independent supplier of specialized hardware items to certain manufacturers that formerly made those items themselves.-----irrelevant
(C) Relatively few manufacturers that start as independent suppliers have been able to expand their business and become direct competitors of the companies they once supplied.----irrelevant
(D) Vernon plans to select the independent suppliers it will use on the basis of submitted bids.------irrelevant
(E) Attending to certain tasks that Vernon performs relatively inefficiently has taken up much of the time and effort of top managers whose time would have been better spent attending to Vernon's core business.-----correct.

作者: w.melhere    时间: 2013-10-22 09:41
1. 31' 这个看题的时候大意了,没有读清选项。 该打该打
2. 34'
3.37'
4.25
5.23' 选错. 这道的解题方向似乎是以前不太知道的. C选项虽然有一定的削弱作用,但较之E与自身的比较就显得不够强了. E: 没有效率的工作占了管理者很多时间,而管理者更应该把时间放在核心产品上. 即,外包出效率较低的产品,管理者就可以更好的管理核心产品,从而提高profit.
作者: cecidefok    时间: 2013-11-1 10:03
精练题
P:五官相异而互补
   五官都有各自的stimuli,并且一个感官的stimuli无法刺激到另一个感官
   有人吃香蕉时说吃起来是蓝色的,或者看到某颜色后说闻到某味
C:这些人的五官是同感的,没有一般人的boundary
问削弱
推测:这些人有幻觉
E.证明是一种病吧............错了

33.
选C

34.
选D,其他好像没什么关系

35.
选C

36.
选E
作者: yuehuasunday    时间: 2013-11-8 16:28
1-9

1.

Background: The 5 senses have been viewed as distinct yet complementary.

Premise: recent research has discovered that some people taste a banana and claim that they are tasting blue.

Conclusion: Synesthesiacs, have senses that do not respect that usual boundaries between the five recognized senses.

(A) Correct

2.

Background: When Caterpillar was born, they move up

Premise: Move in the direction opposite to the pull of gravity

Conclusion: a strong light source was placed at the bottom of a tree, and caterpillars, after hatching, climbed downward

© Correct

3.

Background: For several years, per capita expenditure on prescription drugs in veronica rose by fifteen percent or more annually.

Premise: prohibited drug manufacturers from raising any of their products prices.

Conclusion: per capita expenditure for prescription drugs continued to increase

(A) Correct

4.

Background: The last member of a new extinct species of a european wild bddr called the giant deer lived in irland about 16,00 years ago

Premise: a large hump on its back

Conclusion: Fossil show no hump

© Correct

5.

Background: Out sourcing is the practice of obtaining from an independent supplier a product or service that a company has previously provide for itself.

Premise: Decline in profit, and plans to host its profits by outsourcing those parts

Conclusion: Independent suppliers can provide at a lower cost

(E) Correct





作者: irenetopia    时间: 2013-11-22 11:15
错了2题……人艰不拆


【精练】1:11
9. The five senses have traditionally been viewed as
distinct yet complementary. Each sense is thought to
have its own range of stimuli that are incapable of
stimulating the other senses. However, recent
research has discovered that some people taste a
banana and claim that they are tasting blue, or see a
color and say that it has a specific smell. This shows
that such people, called synesthesiacs, have senses
that do not respect the usual boundaries between the
five recognized senses.
Which one of the following statements, if true, most
seriously weakens the argument?
逻辑链:五个感官是互补的——但是每一个感官并不能影响到其他感官——但是会有一种叫synesthesiacs的人分不清这些——因此,这些人在这几个感官方面是没有遵循界限的。
问削弱:这题我选错了。现在整理思路:前提都是对的,五个都是分开的,S也确实分不清这个。结论S没有分清界限。是找S不具有代表性?这个不仅仅是S才会犯的,而是大家都会犯的。

(A) Synesthesiacs demonstrate a general,
systematic impairment in their ability to use
and understand words.——bingo!这是普遍的错误,不是s才有的

(B) Recent evidence strongly suggests that there
are other senses besides sight, touch, smell,
hearing, and taste.——无关,除了五官外还有别的器官并不影响结果

(C) The particular ways in which sensory
experiences overlap in synesthesiacs follow a
definite pattern.——在S种,感官经历的特殊方式与特定的模式重叠,这个是加强了吧算
(D) The synesthetic phenomenon has been
described in the legends of various cultures.——这个现象是出现在传说里的?好吧……排除
(E) Synesthesiacs can be temporarily rid of their
synesthetic experiences by the use of drugs.——他们可以通过药物来摆脱这个,说的是解决方法,也不是weaken
Weaken Question Problem Set  NO.4




【逻辑链】


33.    (31934-!-item-!-188;#058&005658)


When a caterpillar emerges from the egg on a tree branch, it immediately climbs upward until it finds a leaf bud to eat.  Biologists thought that this behavior displayed an innate tendency to move in the direction opposite to the pull of gravity.  In a recent experiment, a strong light source was placed at the bottom of a tree, and caterpillars, after hatching, climbed downward.

Which of the following hypotheses is best supported by the statements given?
逻辑链:蚜虫在树上孵化后会向上爬找输液吃,生物学家认为是一个跟重力相反的天性导致?——在最近的研究中,发现当在树底部放强光的时候,蚜虫往下爬了
问be support
思路:就是选项中不能出现新信息,从最新的研究来看,蚜虫应该是喜欢往有光的地方跑

(A) Caterpillars have an innate tendency to move in the direction of gravity.——不对,两个情况的反应不同,不能只对一个情况作解释
(B) Newly hatched caterpillars are unable to see in the dark.——无关
(C) Newly hatched caterpillars move towards the strongest light source in the environment.——bingo!很好地说明了
(D) Newly hatched caterpillars move toward the leaf bud nearest to them.——文中没出现
(E) The eyes of newly hatched caterpillars become less sensitive to light over time.——weaken了现象,无关


34.    2:02

For several years, per capita expenditure on prescription drugs in Voronia rose by fifteen percent or more annually.  In order to curb these dramatic increases, the ministry of health prohibited drug manufacturers from raising any of their products' prices.  Even though use of prescription drugs did not expand after this price freeze, per capita expenditure for prescription drugs continued to increase by a substantial percentage each year.

Which of the following, if true, most helps to explain why the ministry's action did not achieve its goal?
逻辑链:这题也错了!V的处方药人均支出增长很快——政府为了禁止酱紫,就出台遏制了价格——即使价格被冻结,人均支出处方药的价格还是每年在涨

问解释:算是方案评估?因为支出增长,所以遏制药价,但是还是在增长。
如果药的价格已经被控制,那么应该是其他的不被控制的药又涨了

(A) After price increases were prohibited, drug manufacturers concentrated on producing new medications to replace existing products.——好牵强的bingo!求指导

(B) The population of Voronia rose steadily throughout the period.——人均和总数没有关系,排除
(C) Improvements in manufacturing processes enabled drug manufacturers to maintain high profit levels on drugs despite the price freeze.——profit和支出也无关,排除
(D) In addition to imposing a price freeze, the government encouraged doctors to prescribe generic versions of common drugs instead of the more expensive brand-name versions.——仅仅说了措施的一个具体内容,排除
(E) After price increases were prohibited, some foreign manufacturers of expensive drugs ceased marketing them in Voronia.——无关


35.    1:40

Which of the following most logically completes the argument?


The last members of a now-extinct species of a European wild deer called the giant deer lived in Ireland about 16,000 years ago.  prehistoric cave paintings in France depict this animal as having a large hump on its back.  Fossils of this animal, however, do not show any hump.  Nevertheless, there is no reason to conclude that the cave paintings are therefore inaccurate in this regard, since __________.
逻辑链:GD是现存的仅有的欧洲野鹿家族成员,16000年轻就存在了。
史前岩画(法国)上的鹿是有hump的,但是这个动物的化石,没有hump。
但是这并不能说明,岩画是不准确的,因为……
思路:也就是岩画是准确的,岩画和化石都是同一种动物,那么为什么不一样,找出岩画和化石相关性,或可以解释这一现象的

(A) some prehistoric cave paintings in France also depict other animals as having a hump——没有相关性
(B) fossils of the giant deer are much more common in Ireland than in France——也是没有相关性
(C) animal humps are composed of fatty tissue, which does not fossilize——bingo!因为此,驼峰不会变成化石
(D) the cave paintings of the giant deer were painted well before 16,000 years ago——这是削弱了题目啊,不是解释
(E) only one currently existing species of deer has any anatomical feature that even remotely resembles a hump——没看懂,所以不要了

36.    1:29

Outsourcing is the practice of obtaining from an independent supplier a product or service that a company has previously provided for itself.  Vernon, Inc., a small manufacturing company that has in recent years experienced a decline in its profits, plans to boost its profits by outsourcing those parts of its business that independent suppliers can provide at lower cost than Vernon can itself.


Which of the following, if true, most strongly supports the prediction that Vernon's plan will achieve its goal?
逻辑链:外包是一个公司找一个独立的供应商来负责一个产品啥的
V公司因为最几年利润下滑,因此计划外包(这些外包公司的成本比他自己做小)
问加强
思路:就是找外包还可以继续增大他利润的,因为它外包的目的是增大利润

(A) Among the parts of its business that Vernon does not plan to outsource are some that require standards of accuracy too high for most independent suppliers to provide at lower cost than Vernon can.——解释了为什么外包利润低,并没有加强
(B) Vernon itself acts as an independent supplier of specialized hardware items to certain manufacturers that formerly made those items themselves.——说他自己也是个外包商……无关
(C) Relatively few manufacturers that start as independent suppliers have been able to expand their business and become direct competitors of the companies they once supplied.——没有从外包商做起的公司可以直面他们曾经服务的公司作为竞争对手无关
(D) Vernon plans to select the independent suppliers it will use on the basis of submitted bids.——weaken了
(E) Attending to certain tasks that Vernon performs relatively inefficiently has taken up much of the time and effort of top managers whose time would have been better spent attending to Vernon's core business.——bingo!这样就可以术业有专攻,profit会上升!
作者: goodluck22    时间: 2014-3-4 16:38
Background: five senses have viewed as distinct
Premise: each sense is thought to have its own range , incapable to stimulate others.
Conclusion: This show such people called synth have sense that don’t repect the usual boundaries
Prephrase: this people do show
A.        correct
B.        Out of scope
C.        Irrelevant
D.        Out of scope
E.        Irrelevant

Background: caterpillar climbs upward until it find a leaf bud to eat
Premise: its behavior displayed in the direction opposite to gravity
A strong light placed at the bottom after hatching climb downward
Prephrase: caterpillar is attracted by the light
A.        OPPOSITE
B.        Irrelevant
C.        Correct
D.        Then don’t explain why
E.        Additional background

Background: per capital on drugs rose by 15 or more
Premise: prohibited drug from raising product price
Concluion: use of drugs did not expand, per capital increase
Prephrase: less people use drugs or new drugs
A.        Correct
B.        Not explain
C.        Profit not the case here
D.        Just some background
E.        Then don’t explain why drug price per person will increase\

Background: last members of a deer lived in Ireland 16000ago
Premise: painting have large hump on back
Fossil of animal don’t have
Conclusion: There is no reason to conclude that cave painting are inaccurate
Prephrase: hump will destroy
A.        other animals out of scope
B.        background
C.        correct
D.        Irrelevant
E.        Irrelevant

Premise: Vernon has experienced a decline in its profit plans to boost by outsourcing
Prephrase: no transition cost or additional cost
A.        Irrelevant
B.        Background
C.        Boost the profit like short term goal not good as E
D.        Irrelvant
E.        COrrect


作者: cyndichiang    时间: 2014-4-8 15:24
精炼 30‘’
BG: The five senses have traditionally been viewed as distinct yet complementary. Each sense is thought to
have its own range of stimuli that are incapable of stimulating the other senses
premise:recent research has discovered that some people taste a banana and claim that they are tasting blue, or see a color and say that it has a specific smell.
Conclusion:people, called synesthesiacs, have senses that do not respect the usual boundaries between the five recognized senses.
Q:Weaken
prephrase: 因的特点:比如说他们看到颜色是因为他们呢脑子里面想到了某种东西,是那种东西的气味
或者是其他因素导致的结果,不知道了。。- -看选项吧
(A) Synesthesiacs demonstrate a general,
systematic impairment in their ability to use
and understand words.------ Correct,我选错了。如果是因为他们语言有问题,那么他们在表述的时候就有问题,因此不是他们呢分不清五个感官
(B) Recent evidence strongly suggests that there
are other senses besides sight, touch, smell,
hearing, and taste.----irrelevant,文章中没有提及其他感觉会对结论造成什么影响
(C) The particular ways in which sensory
experiences overlap in synesthesiacs follow a
definite pattern.----我错选了C,感觉是这样的,不管因为是那种模式的Synesthesiacs,S 这个事实已经在了,所以解释结论而没有说到原因,对因推果的程度没有削弱。
(D) The synesthetic phenomenon has been
described in the legends of various cultures.---irrelevant,与various cultures无关
(E) Synesthesiacs can be temporarily rid of their
synesthetic experiences by the use of drugs.----irrelevant,this information does not infer that drugs have what kind of effect on 5 senses,thus those mistakenly identify 5 senses still are Synesthesiacs.


逻辑链:
1. 25''
BG:When a caterpillar emerges from the egg on a tree branch, it immediately climbs upward until it finds a leaf bud to eat.  Biologists thought that this behavior displayed an innate tendency to move in the direction opposite to the pull of gravity.  
premsie: In a recent experiment, a strong light source was placed at the bottom of a tree, and caterpillars, after hatching, climbed downward
Conclusion:???
Q:Must be true
prephrase: The conclusion that the caterpillar displayed an innate tendency to move in the direction opposite to the pull of gravity is wrong.


3.36''
premise: the ministry of health prohibited drug manufacturers from raising any of their products' prices.  premise:use of prescription drugs did not expand after this price freeze
Conclusion:per capita expenditure for prescription drugs continued to increase by a substantial percentage each year.
Q:Explain:
prephrase: (price * number)/population=per capita expenditure for prescription drugs
人数变了?


4.29‘’
premise:cave paintings in France depict wild deer as having a large hump on its back.  
premise:Fossils of this animal do not show any hump
Conclusion: the cave paintings are inaccurate in this regard.
Q:weaken.(我觉得因为问why there is no reason to conclude,也就是削弱这个结论,就这么理解好了,待会看选项。。)
prephrase: 因的一个特点导致不同的结果:fossils在挖掘的时候丢失了某一部分,fossils在几千年下来把hump弄没了


5.36‘’
premise:  independent suppliers can provide at lower cost than Vernon can itself.
Conclusion: by outsourcing, the company can boost its profit.
Q: support
prephrase: 因果有联系的例证:outsourcing can help cut cost in some way
               否定一个方案副作用:outsourcing cannot hurt company's revenue
              

作者: simonleo    时间: 2014-5-2 12:03
ACACE
1.
Time: 31s
P:five sense viewed as distict but complementary, each sense is thought to have its own range of stimuli that are incapable of stimulating the other senses.
C-P: recent research show some people eat banana and claim that they are blue, or see color and say it has a specific smell.
C: such people have senses that don't respect the usual boundaries between the five recognized senses.
Pre:削弱。只要说明了其实是有boundaries的即可。
(A) Synesthesiacs demonstrate a general,
systematic impairment in their ability to use
and understand words. 正解,说明不是他们没有boundaries,而是因为不会表达和理解词语的含义。
(B) Recent evidence strongly suggests that there
are other senses besides sight, touch, smell,
hearing, and taste. 无关,要考虑的是这五种感觉之间有没有boundaries。
(C) The particular ways in which sensory
experiences overlap in synesthesiacs follow a
definite pattern. 加强
(D) The synesthetic phenomenon has been
described in the legends of various cultures. 不能削弱吗,一直都有描述与他们是不是真的没有boundaries无关
(E) Synesthesiacs can be temporarily rid of their
synesthetic experiences by the use of drugs. 不能削弱
刚好说明这是有boundaries的

2.
Time:36s
P:when a caterpillar emerges from the egg on a tree branch, it climbs upward until it finds a leaf bud to eat.
Biologists thought that this behavior displayed an innate tendency to move in the direction opposite to gravity.
strong light source was placed at the bottom of a tree, the caterpillars climed downward.
Pre:must be true.毛毛虫是有向阳性的。
(A) Caterpillars have an innate tendency to move in the direction of gravity. 推不出
(B) Newly hatched caterpillars are unable to see in the dark. 推不出
(C) Newly hatched caterpillars move towards the strongest light source in the environment. 正解,可以推出
(D) Newly hatched caterpillars move toward the leaf bud nearest to them. 推不出是不是向nearest的靠近
(E) The eyes of newly hatched caterpillars become less sensitive to light over time. 推不出

3.
P:per capita expenditure on prescripition drugs rose.
in order to curb the dramatic increases, the ministry of health prohibited drug manufacturers from raising any of their products' prices.
Even though use of prescription drugs did not expand after this price freeze, per capita expenditure for prescription drugs continued to increase.
Pre:solve the paradox.也许是禁止了以后医生开了更贵的药。
(A) After price increases were prohibited, drug manufacturers concentrated on producing new medications to replace existing products. 正解,相比较下最好的选项,开发的新药可能就比以前更贵一些
(B) The population of Voronia rose steadily throughout the period.人口增多与人均花费无关
(C) Improvements in manufacturing processes enabled drug manufacturers to maintain high profit levels on drugs despite the price freeze.但是不能解释为什么人均消费增多
(D) In addition to imposing a price freeze, the government encouraged doctors to prescribe generic versions of common drugs instead of the more expensive brand-name versions.更不能解释了
(E) After price increases were prohibited, some foreign manufacturers of expensive drugs ceased marketing them in Voronia.不能解释,贵的药品没了按道理不会增加啊

4.
Time:24s
P:paintings show the animal as having a large hump on its back.
C-P:fossils of the animal,however, do not show any hump.
C: no reason to conclude that the cave paintings are inaccurate.
Pre:fill in the blank_assumption.也许驼峰是没有骨骼的或者骨骼已经消失了,不显现在化石中了。
(A) some prehistoric cave paintings in France also depict other animals as having a hump 无关
(B) fossils of the giant deer are much more common in Ireland than in France无关
(C) animal humps are composed of fatty tissue, which does not fossilize 正解 不需要取非都很明显,取非的话 说humps 会 fossilize,那么那些化石是可以说明壁画是不准确的。
(D) the cave paintings of the giant deer were painted well before 16,000 years ago无关
(E) only one currently existing species of deer has any anatomical feature that even remotely resembles a hump 无关

5.
Time:51s
B:Outsourcing is the practice of obtaining from an independent supplier a product or service that a company has previously provided for itself.
P:V experienced a decline in profits, plan to boost its profits by outsourcing those parts of its business that independent suppliers can provide at lower cost than can itself.
Pre:C-->Eutsouring-->boost profits.

(A) Among the parts of its business that Vernon does not plan to outsource are some that require standards of accuracy too high for most independent suppliers to provide at lower cost than Vernon can. 无法加强,不需要考虑不被替代的部分
(B) Vernon itself acts as an independent supplier of specialized hardware items to certain manufacturers that formerly made those items themselves. 无关
(C) Relatively few manufacturers that start as independent suppliers have been able to expand their business and become direct competitors of the companies they once supplied.无关
(D) Vernon plans to select the independent suppliers it will use on the basis of submitted bids. 无关
(E) Attending to certain tasks that Vernon performs relatively inefficiently has taken up much of the time and effort of top managers whose time would have been better spent attending to Vernon's core business. 正解,当top managers专心在core的时候可以time effort可以better spent则利润是可以增加的。
作者: 采药去    时间: 2014-10-25 18:06

我选的B ...精练题 又错了 啊啊啊啊啊啊啊啊

BG:五个区域分开却互补,各有各的region
P:有些人吃香蕉却说they are tasting bule;看见一个颜色却说它有奇特的味道
C:这些人don't repect (遵守) usual bountries between the five senses
问weaken
A.S综合征证明他们their ability to use and understand words 有损伤
B. 除了5个sense 有其他sense存在
C.无关
D.无关
E.用药来避免--无关


A:it may not be that their senses overlap but rather that they lack the ability to properly
express themselves. Thus, their claim to taste a banana and see blue might not be a reflection of that
actually occurring but rather a reflection of the words they use to describe taste.


B:
Unfortunately, that judgment is not
supported by the answer choice. Not enough information is provided by the answer choice to say what
role, if any, is played by these other senses.

逻辑链:
33.resolve the paradox
situation1: after hatch they climbs  upward
situation 2: when there is a strong light at the bottom , they climbed downward
why?
C

34.
BG: 每人的处方药花费每年上升很多, 政府禁止药商涨价
用药量没有变多,但是 每人的处方药花费还是每年上升很多
why
推测: 处方药用药量*价格/人数  人数可能下降

A.用新药代替以前的药--correct(那其实还是没有变啊,还是和以前一样啊, 该长还是长啊)
B.weaken
C.irrelevant
D.很迷惑啊 weaken 价格下降了
E.irrelevant

35.
P:画上有hump, 化石里没有humps
C: 不能说画里的不准确
推测 找个理由解释humps消失原因就可以了

C

36.就是在c和E里徘徊 结果还是选错了
BG: 外包就是把以前拿来自己产的拿给别人产
P:V外包了部分业务(这种业务比自己产更lower cost)
C: 这个plan 会成功的

support
A.weaken
B.irrelevant
C.很少的idependent suppliers 可以扩展他们的业务并成为竞争者(就是indepent supplier的坏处)
D.irrelevant
E.让manager有更多的时间在core business 上

其实C 还是比较不准确 就算V 以前是自己产,但是也不代表他是indepent supplier啊

作者: spencerchan    时间: 2015-8-14 15:11
41''
B: five senses, each sense would not stimulate the other senses
P: People taste a banana + see a color= different taste
C: Those people do not respect the usual boundaries between the five senses
Prep: Even having usual boundaries, people’s sense still be influenced by other senses
A: Strengthen
B: Irrelevant
C: CORRECT
D: Irrelevant/ Strengthen
E: Irrelevant

30’’
P: Strong light place at bottom
C: Bird climb downward
Prep: The sunlight/ strong light led bird’s climbing direction, it is not innate reaction to gravity
A: Weaken, and it cannot be explained
B: Irrelevant
C: CORRECT
D: Weaken
E: Irrelevant

40’’
P: Price freeze
C: the quantity of sold drugs has not change,but per capita expenditure increase
Prep: People use another expensive drugs??
A: CORRECT?
B: Irrelevant
C: Irrelevant
D: Weaken
E: Weaken

27’’
P: Traditional painting shows the giant deer had a large hump
P: Fossil shows the giant deer had not a large hump
C: The tradition painting also is right
Prep: The feather of giant deer would change with time
A: Irrelevant
B: Irrelevant
C: CORRECT
D: Irrelevant
E: Weaken

25’’
P: Outsourcing the business that independent suppliers can provide at lower cost
C: Profit will increase
Prep: Outsourcing will decrease the company’s profit, so the option should eliminate the possibility
A: Irrelevant
B: Irrelevant
C: Weaken
D: Irrelevant
E: CORRECT

作者: MIA926    时间: 2015-9-1 08:47
20150901CR小分队
【精练】
9. The five senses have traditionally been viewed as distinct yet complementary. Each sense is thought to have its own range of stimuli that are incapable of stimulating the other senses. However, recent research has discovered that some people taste a banana and claim that they are tasting blue, or see a color and say that it has a specific smell. This shows that such people, called synesthesiacs, have senses that do not respect the usual boundaries between the five recognized senses.
计时:30.30s
逻辑链:本来认为五感之间是相互独立的,但是最近发现有人说吃的东西是蓝的的感觉,看颜色看到了味道,所以说这种人的五感之间没有正常的界限
推测:首先出现research就很可能有gap了…然后很可能是这些人脑洞太大或者表达有问题吧…
Which one of the following statements, if true, most seriously weakens the argument?
(A) Synesthesiacs demonstrate a general,
systematic impairment in their ability to use
and understand words. 正确
(B) Recent evidence strongly suggests that there
are other senses besides sight, touch, smell,
hearing, and taste. 无关
(C) The particular ways in which sensory
experiences overlap in synesthesiacs follow a
definite pattern. 加强
(D) The synesthetic phenomenon has been
described in the legends of various cultures.
(E) Synesthesiacs can be temporarily rid of their
synesthetic experiences by the use of drugs. 无关
Weaken Question Problem Set  NO.4




【逻辑链】


33.    (31934-!-item-!-188;#058&005658)
When a caterpillar emerges from the egg on a tree branch, it immediately climbs upward until it finds a leaf bud to eat.  Biologists thought that this behavior displayed an innate tendency to move in the direction opposite to the pull of gravity.  In a recent experiment, a strong light source was placed at the bottom of a tree, and caterpillars, after hatching, climbed downward.

逻辑链:本来发现毛毛虫一出来就往上爬找吃的,然后觉得他们有反地心引力的倾向,但是最近发现在树底打光他们就往下爬
Which of the following hypotheses is best supported by the statements given?
(A) Caterpillars have an innate tendency to move in the direction of gravity. 无关
(B) Newly hatched caterpillars are unable to see in the dark. 无关
(C) Newly hatched caterpillars move towards the strongest light source in the environment. 正确
(D) Newly hatched caterpillars move toward the leaf bud nearest to them. 无关
(E) The eyes of newly hatched caterpillars become less sensitive to light over time. 无关


34.    (31982-!-item-!-188;#058&005681)
For several years, per capita expenditure on prescription drugs in Voronia rose by fifteen percent or more annually.  In order to curb these dramatic increases, the ministry of health prohibited drug manufacturers from raising any of their products' prices.  Even though use of prescription drugs did not expand after this price freeze, per capita expenditure for prescription drugs continued to increase by a substantial percentage each year.
逻辑链:几年来人均的处方药花费都很高,然后政府下令禁止生产商涨价,但是价格还是在上涨
推测:处方药啊,可能是开药的医生坑钱呢,然后政策可能会带来不良后果
Which of the following, if true, most helps to explain why the ministry's action did not achieve its goal?
(A) After price increases were prohibited, drug manufacturers concentrated on producing new medications to replace existing products. 正确
(B) The population of Voronia rose steadily throughout the period. 无关 人均
(C) Improvements in manufacturing processes enabled drug manufacturers to maintain high profit levels on drugs despite the price freeze. 无关
(D) In addition to imposing a price freeze, the government encouraged doctors to prescribe generic versions of common drugs instead of the more expensive brand-name versions. 加强政策
(E) After price increases were prohibited, some foreign manufacturers of expensive drugs ceased marketing them in Voronia. 加强政策


35.    (32076-!-item-!-188;#058&006018)
Which of the following most logically completes the argument?
The last members of a now-extinct species of a European wild deer called the giant deer lived in Ireland about 16,000 years ago.  Prehistoric cave paintings in France depict this animal as having a large hump on its back.  Fossils of this animal, however, do not show any hump.  Nevertheless, there is no reason to conclude that the cave paintings are therefore inaccurate in this regard, since __________.
逻辑链:壁画上画的鹿背上有坨包但是化石发现并没有,但也不能说人家画错了
推测:有包但不一定是骨头啊…

(A) some prehistoric cave paintings in France also depict other animals as having a hump 削弱…
(B) fossils of the giant deer are much more common in Ireland than in France 无关
(C) animal humps are composed of fatty tissue, which does not fossilize 正确
(D) the cave paintings of the giant deer were painted well before 16,000 years ago 无关
(E) only one currently existing species of deer has any anatomical feature that even remotely resembles a hump 无关


36.    (32124-!-item-!-188;#058&006025)
Outsourcing is the practice of obtaining from an independent supplier a product or service that a company has previously provided for itself.  Vernon, Inc., a small manufacturing company that has in recent years experienced a decline in its profits, plans to boost its profits by outsourcing those parts of its business that independent suppliers can provide at lower cost than Vernon can itself.
逻辑链:外包,然后一个小公司最近利润下降所以要外包了,cost少,为了增加利润

Which of the following, if true, most strongly supports the prediction that Vernon's plan will achieve its goal?
(A) Among the parts of its business that Vernon does not plan to outsource are some that require standards of accuracy too high for most independent suppliers to provide at lower cost than Vernon can. 无关
(B) Vernon itself acts as an independent supplier of specialized hardware items to certain manufacturers that formerly made those items themselves. 无关
(C) Relatively few manufacturers that start as independent suppliers have been able to expand their business and become direct competitors of the companies they once supplied. 无关
(D) Vernon plans to select the independent suppliers it will use on the basis of submitted bids. 无关
(E) Attending to certain tasks that Vernon performs relatively inefficiently has taken up much of the time and effort of top managers whose time would have been better spent attending to Vernon's core business. 正确 增加了外包的好处

作者: Geranium_Bayy    时间: 2016-3-29 21:06
【精练】
(做错)9. The five senses have traditionally been viewed as distinct yet complementary. Each sense is thought to have its own range of stimuli that are incapable of stimulating the other senses. However, recent research has discovered that some people taste a banana and claim that they are tasting blue, or see a color and say that it has a specific smell. This shows that such people, called synesthesiacs, have senses that do not respect the usual boundaries between the five recognized senses.
Which one of the following statements, if true, most seriously weakens the argument?
(A) Synesthesiacs demonstrate a general, systematic impairment in their ability to use and understand words.
(B) Recent evidence strongly suggests that there are other senses besides sight, touch, smell, hearing, and taste.
(C) The particular ways in which sensory experiences overlap in synesthesiacs follow a definite pattern.
(D) The synesthetic phenomenon has been described in the legends of various cultures.
(E) Synesthesiacs can be temporarily rid of their synesthetic experiences by the use of drugs.
做题总共时间:1’41’’

33.   When a caterpillar emerges from the egg on a tree branch, it immediately climbs upward until it finds a leaf bud to eat.  Biologists thought that this behavior displayed an innate tendency to move in the direction opposite to the pull of gravity.  In a recent experiment, a strong light source was placed at the bottom of a tree, and caterpillars, after hatching, climbed downward.
Which of the following hypotheses is best supported by the statements given?
(A) Caterpillars have an innate tendency to move in the direction of gravity.
(B) Newly hatched caterpillars are unable to see in the dark.
(C) Newly hatched caterpillars move towards the strongest light source in the environment.
(D) Newly hatched caterpillars move toward the leaf bud nearest to them.
(E) The eyes of newly hatched caterpillars become less sensitive to light over time.


34. For several years, per capita expenditure on prescription drugs in Voronia rose by fifteen percent or more annually.  In order to curb these dramatic increases, the ministry of health prohibited drug manufacturers from raising any of their products' prices.  Even though use of prescription drugs did not expand after this price freeze, per capita expenditure for prescription drugs continued to increase by a substantial percentage each year.
Which of the following, if true, most helps to explain why the ministry's action did not achieve its goal?
(A) After price increases were prohibited, drug manufacturers concentrated on producing new medications to replace existing products.
(B) The population of Voronia rose steadily throughout the period. Even though the population is increasing, the use did not expand after this prize freeze
(C) Improvements in manufacturing processes enabled drug manufacturers to maintain high profit levels on drugs despite the price freeze. The price is not change, so that the expenditure is not change too. Expenditure 不等于 profit levels
(D) In addition to imposing a price freeze, the government encouraged doctors to prescribe generic versions of common drugs instead of the more expensive brand-name versions. 错误
(E) After price increases were prohibited, some foreign manufacturers of expensive drugs ceased marketing them in Voronia. Weaken, 那么expenditure就减少了

35.    (32076-!-item-!-188;#058&006018)
Which of the following most logically completes the argument?
The last members of a now-extinct species of a European wild deer called the giant deer lived in Ireland about 16,000 years ago.  rehistoric cave paintings in France depict this animal as having a large hump on its back.  Fossils of this animal, however, do not show any hump.  Nevertheless, there is no reason to conclude that the cave paintings are therefore inaccurate in this regard, since __________.
(A) some prehistoric cave paintings in France also depict other animals as having a hump
(B) fossils of the giant deer are much more common in Ireland than in France
(C) animal humps are composed of fatty tissue, which does not fossilize
(D) the cave paintings of the giant deer were painted well before 16,000 years ago
(E) only one currently existing species of deer has any anatomical feature that even remotely resembles a hump

36.    (32124-!-item-!-188;#058&006025)
Outsourcing is the practice of obtaining from an independent supplier a product or service that a company has previously provided for itself.  Vernon, Inc., a small manufacturing company that has in recent years experienced a decline in its profits, plans to boost its profits by outsourcing those parts of its business that independent suppliers can provide at lower cost than Vernon can itself.
Which of the following, if true, most strongly supports the prediction that Vernon's plan will achieve its goal?
(A) Among the parts of its business that Vernon does not plan to outsource are some that require standards of accuracy too high for most independent suppliers to provide at lower cost than Vernon can.
(B) Vernon itself acts as an independent supplier of specialized hardware items to certain manufacturers that formerly made those items themselves.
(C) Relatively few manufacturers that start as independent suppliers have been able to expand their business and become direct competitors of the companies they once supplied.
(D) Vernon plans to select the independent suppliers it will use on the basis of submitted bids.
(E) Attending to certain tasks that Vernon performs relatively inefficiently has taken up much of the time and effort of top managers whose time would have been better spent attending to Vernon's core business.
做题时间:1’25’’

作者: fesche    时间: 2016-5-2 06:14
five senses are separated
recently some people say banana tastes blue, and color has smell
C: some people’s five senses do not respect the boundaries
weaken
GAP: people imagination, not the real taste or smell
A

P: caterpillar climbed up ever since they were born
ability to oppose gravity
study shows lights in bottom of the trees and caterpillar will climb downwards
C

P: expenditure arises
price doesn’t not arise
expenditure continue to arise
explain
more quantity
D-A

C

a company profit declines
want to outsource some service
support
outsource is cheaper
deliver same outcome
E
作者: 宋痂岛__    时间: 2017-12-11 16:24
ACDCE
错了一道
作者: May97    时间: 2018-1-22 09:08
9. T:37S
BG: The five senses have traditionally been viewed as distinct yet complementary. Each sense is thought to have its own range of stimuli that are incapable of stimulating the other senses.
P:However, recent research has discovered that some people taste a banana and claim that they are tasting blue, or see a color and say that it has a specific smell.
C: This shows that such people, called synesthesiacs, have senses that do not respect the usual boundaries between the five recognized senses.
Weaken:有其他的理由可以解释这种想象的产生
(A) Synesthesiacs demonstrate a general, systematic impairment in their ability to use and understand words.--->correct
(B) Recent evidence strongly suggests that there are other senses besides sight, touch, smell, hearing, and taste.--->irreverent
(C) The particular ways in which sensory experiences overlap in synesthesiacs follow a definite pattern.--->irreverent
(D) The synesthetic phenomenon has been described in the legends of various cultures.--->irreverent
(E) Synesthesiacs can be temporarily rid of their synesthetic experiences by the use of drugs.--->irreverent

33:T:34s
When a caterpillar emerges from the egg on a tree branch, it immediately climbs upward until it finds a leaf bud to eat.  
Biologists thought that this behavior displayed an innate tendency to move in the direction opposite to the pull of gravity.  
In a recent experiment, a strong light source was placed at the bottom of a tree, and caterpillars, after hatching, climbed downward.
Find hypotheses:light与C的移动方向有关
(A) Caterpillars have an innate tendency to move in the direction of gravity.--->与第三句话相反
(B) Newly hatched caterpillars are unable to see in the dark.--->irreverent
(C) Newly hatched caterpillars move towards the strongest light source in the environment.--->correct
(D) Newly hatched caterpillars move toward the leaf bud nearest to them.--->irreverent
(E) The eyes of newly hatched caterpillars become less sensitive to light over time.--->irreverent

34. T:38s
BG: For several years, per capita expenditure on prescription drugs in Voronia rose by fifteen percent or more annually.  
C: In order to curb these dramatic increases, the ministry of health prohibited drug manufacturers from raising any of their products' prices.  
P: Even though use of prescription drugs did not expand after this price freeze, per capita expenditure for prescription drugs continued to increase by a substantial percentage each year.
E: 没有思路
(A) After price increases were prohibited, drug manufacturers concentrated on producing new medications to replace existing products.--->correct
(B) The population of Voronia rose steadily throughout the period.--->pcd减少
(C) Improvements in manufacturing processes enabled drug manufacturers to maintain high profit levels on drugs despite the price freeze.--->pcd增长依然得到遏制
(D) In addition to imposing a price freeze, the government encouraged doctors to prescribe generic versions of common drugs instead of the more expensive brand-name versions.--->pcd降低
(E) After price increases were prohibited, some foreign manufacturers of expensive drugs ceased marketing them in Voronia.--->pcd降低

35. T:34s
BG:The last members of a now-extinct species of a European wild deer called the giant deer lived in Ireland about 16,000 years ago.  
P:rehistoric cave paintings in France depict this animal as having a large hump on its back.  
P:Fossils of this animal, however, do not show any hump.  
C:Nevertheless, there is no reason to conclude that the cave paintings are therefore inaccurate in this regard, since __________.
Support:由于某些原因使得洞中的画像和化石产生了差异
(A) some prehistoric cave paintings in France also depict other animals as having a hump--->irreverent
(B) fossils of the giant deer are much more common in Ireland than in France--->irreverent
(C) animal humps are composed of fatty tissue, which does not fossilize--->correct
(D) the cave paintings of the giant deer were painted well before 16,000 years ago--->weaken
(E) only one currently existing species of deer has any anatomical feature that even remotely resembles a hump--->irreverent

36. T: 36s
BG:Outsourcing is the practice of obtaining from an independent supplier a product or service that a company has previously provided for itself.  
P:Vernon, Inc., a small manufacturing company that has in recent years experienced a decline in its profits,
C;plans to boost its profits by outsourcing those parts of its business that independent suppliers can provide at lower cost than Vernon can itself.
Support:P=R-C, C下降的同时,R要保持不变或者上升
(A) Among the parts of its business that Vernon does not plan to outsource are some that require standards of accuracy too high for most independent suppliers to provide at lower cost than Vernon can.--->irreverent
(B) Vernon itself acts as an independent supplier of specialized hardware items to certain manufacturers that formerly made those items themselves.--->irreverent
(C) Relatively few manufacturers that start as independent suppliers have been able to expand their business and become direct competitors of the companies they once supplied.--->irreverent
(D) Vernon plans to select the independent suppliers it will use on the basis of submitted bids.--->irreverent
(E) Attending to certain tasks that Vernon performs relatively inefficiently has taken up much of the time and effort of top managers whose time would have been better spent attending to Vernon's core business.--->correct
作者: ba1a糖儿    时间: 2018-7-18 15:56
"P outsource
C boost profit
BI recent years a decline in its profit
A 不外包出去的是高精尖。但是与profit没关系
B 与外包没关系
c 不怕倒打一耙,profit有保障
D bid和profit无关?
E support了公司的决定,但是不support可以boost profit的结果

PREP
外包part of its product->↓cost,↑profit
(C) Relatively few manufacturers that start as independent suppliers have been able to expand their business and become direct competitors of the companies they once supplied.
C 就算有outsource,没有多少外包商可以变成competitor
可能会对profit产生影响,先留着
(E) Attending to certain tasks that Vernon performs relatively inefficiently has taken up much of the time and effort of top managers whose time would have been better spent attending to Vernon's core business.
E 时间效率筹划的改变使得V的efficiency可以提高

把 part of its work外包,
part外包的情况下,就像苹果,研发不会外包吧?只是组装和生产需要外包,而真正值钱的部分就在研发上。说明重要的部分是不会外包的!!!prep题中的C在这种情况下无须担心。

我第一次做prep的这道题也选了C。。细想一下,就算被competitor抢占了生意,对profit会有影响吗?还是来看苹果的例子.. 零部件非正版卖的很多啊,但是核心技术没被盗取,所以影响没那么大吧

=。=这个例子可能扯远了,主要想呈现两个题目的不同.. .. 至于prep题CE的对比.. ..
"

Which of the following, if true, most strongly supports the prediction that Vernon's plan will achieve its goal?
目的就是boost profit,更多的是对结论的补充E,而非排除其他可能C(是不是support的情况下就是填gap而strengthen就是排除其他可能啊??求大佬指点)
作者: Akiii    时间: 2018-9-6 09:48
第三题的人均药物支出,居然分子不是用药的价格,而是市场上所有发行的总药物价格……不管你有没有买,只要发了新药,就增加了总药物的价格。。。
new drugs are not subject to the price controls (since there is no current price at which they can be frozen - the price controls only apply to drugs that already have a price). therefore, these new drugs could indeed account for increased costs, even if consumers are not purchasing any more drugs than they did before.
作者: echo-LUO    时间: 2018-9-17 14:57
9. 精炼
background information: people's five senses are distinct, each has its own range of stimuli
premise: some people see a color say that it has smell
conclusion: these people's senses do not respect the usual boundaries
猜想:并不是同一种sense感受到了两种,而是同时多种感觉作用
(A) Synesthesiacs demonstrate a general, systematic impairment in their ability to use and understand words.
(B) Recent evidence strongly suggests that there are other senses besides sight, touch, smell, hearing, and taste.
(C) The particular ways in which sensory experiences overlap in synesthesiacs follow a definite pattern.
(D) The synesthetic phenomenon has been described in the legends of various cultures.
(E) Synesthesiacs can be temporarily rid of their synesthetic experiences by the use of drugs.

选项B:有证据表示,人不仅只有五感。所以可能是另一种senses产生这种现象
正确选项:A  说明S患者是感官损坏,因此不能得出结论说感官相通
错误原因:选项B在题目中没有支撑的依据,也没有任何材料提及其他的感官与这个现象有关,只是“可能”




33. 逻辑链
caterpillar climb upward to find food--->biologists: display an innate tendency to move in the direction opposite to the pull of gravity---->转折: when a strong light source at the bottom of a tree, the caterpiller climbed downward
猜想:没有所谓的innate tendency, 而是趋光性和趋热性
(A) Caterpillars have an innate tendency to move in the direction of gravity.
(B) Newly hatched caterpillars are unable to see in the dark.
(C) Newly hatched caterpillars move towards the strongest light source in the environment.
(D) Newly hatched caterpillars move toward the leaf bud nearest to them.
(E) The eyes of newly hatched caterpillars become less sensitive to light over time.


选项C:caterpillar是向最强光源移动


34.
background: spend on drugs increase--->in order to curb the increase, the government ask drug manufacturers not raise price
premise: the use of the drug didn't expand after the price freeze
conclusion: spend still increase
(A) After price increases were prohibited, drug manufacturers concentrated on producing new medications to replace existing products.
(B) The population of Voronia rose steadily throughout the period.
(C) Improvements in manufacturing processes enabled drug manufacturers to maintain high profit levels on drugs despite the price freeze.
(D) In addition to imposing a price freeze, the government encouraged doctors to prescribe generic versions of common drugs instead of the more expensive brand-name versions.
(E) After price increases were prohibited, some foreign manufacturers of expensive drugs ceased marketing them in Voronia.

选项B:V患者增加,购买增多
正确选项:A 意思理解错误,V是一种灌木,V增加,成本降低,与题目相反。所以应该是,生产商看不能提价,就研制新产品来增加销量


35.
background information: the fossil of g deer shows that it has no hump, but the painting of g deer show a hump.
premise: 选项
conclusion: but, the painting is not inaccurate
(A) some prehistoric cave paintings in France also depict other animals as having a hump
(B) fossils of the giant deer are much more common in Ireland than in France
(C) animal humps are composed of fatty tissue, which does not fossilize
(D) the cave paintings of the giant deer were painted well before 16,000 years ago
(E) only one currently existing species of deer has any anatomical feature that even remotely resembles a hump
选项C: 动物有hump是肥胖症状的表现,所以可能是画上的deer是只胖deer



36.
Outsourcing is the practice of obtaining from an independent supplier a product or service that a company has previously provided for itself.  Vernon, Inc., a small manufacturing company that has in recent years experienced a decline in its profits, plans to boost its profits by outsourcing those parts of its business that independent suppliers can provide at lower cost than Vernon can itself.
background information: what is outsourcing
premise: V company want to outsource their unprofitable parts
conclusion: success
(A) Among the parts of its business that Vernon does not plan to outsource are some that require standards of accuracy too high for most independent suppliers to provide at lower cost than Vernon can.
(B) Vernon itself acts as an independent supplier of specialized hardware items to certain manufacturers that formerly made those items themselves.
(C) Relatively few manufacturers that start as independent suppliers have been able to expand their business and become direct competitors of the companies they once supplied.
(D) Vernon plans to select the independent suppliers it will use on the basis of submitted bids.
(E) Attending to certain tasks that Vernon performs relatively inefficiently has taken up much of the time and effort of top managers whose time would have been better spent attending to Vernon's core business.

选项E:说明外包之后,顶尖的人员将集中在核心商务上





作者: 喜皮    时间: 2018-12-14 14:27
1.--->45s**
premise:some people eat banana , they say they are tasting blue and when seeing a color they smell odour.
conclusion:some people who has synthes...dont have usual sense that have boundary
bg:the research stated that the five senses are independent from each other so boundary is available among them.
prephrase:weaken--->【sign a thing/Z/accident or misunderstanding】their senses are actually have boundary/another reason cause that feeling but not because of no boundary/that feeling is a kind of misunderstanding.

(A) Synesthesiacs demonstrate a general,
systematic impairment in their ability to use
and understand words.
--->irrelevant
(B) Recent evidence strongly suggests that there
are other senses besides sight, touch, smell,
hearing, and taste.
--->right.another reason cause that feeling
(C) The particular ways in which sensory
experiences overlap in synesthesiacs follow a
definite pattern.
--->irrelevant
(D) The synesthetic phenomenon has been
described in the legends of various cultures.
--->irrelevant
(E) Synesthesiacs can be temporarily rid of their
synesthetic experiences by the use of drugs.
--->irrelevant
OA:A

The conclusion is in the last sentence, that some people “have senses that do not respect the usual boundaries between the five recognized senses.”Instead of keeping their senses distinct, these individuals have an overlap.
A的解释: This is the correct answer. If the synesthesiacs have a systematic impairment in their use of language it may not be that their senses overlap but rather that they lack the ability to properlyexpress themselves. Thus, their claim to taste a banana and see blue might not be a reflection of thatactually occurring but rather a reflection of the words they use to describe taste. 因为语言表达能力受损,所以无法清楚的表达出自己的意思,因此他们说的话没有很大的参考价值。---->他因……但是个人感觉这个选项也很模糊
但是B题干里没有说明他们的感觉与新的感觉有关/Manhattan解释:B攻击了背景,所以是out of scope
作者: 喜皮    时间: 2018-12-14 14:53
2.--->42s
A) Caterpillars have an innate tendency to move in the direction of gravity.
(B) Newly hatched caterpillars are unable to see in the dark.
(C) Newly hatched caterpillars move towards the strongest light source in the environment.
(D) Newly hatched caterpillars move toward the leaf bud nearest to them.
(E) The eyes of newly hatched caterpillars become less sensitive to light over time.
C

3.--->45s(?)
(A) After price increases were prohibited, drug manufacturers concentrated on producing new medications to replace existing products.
(B) The population of Voronia rose steadily throughout the period.
(C) Improvements in manufacturing processes enabled drug manufacturers to maintain high profit levels on drugs despite the price freeze.
(D) In addition to imposing a price freeze, the government encouraged doctors to prescribe generic versions of common drugs instead of the more expensive brand-name versions.
(E) After price increases were prohibited, some foreign manufacturers of expensive drugs ceased marketing them in Voronia.
A
STACEY:
"per-capita expenditure" is the total price paid for drugs (price per pill * # of pills) divided by the number of people.

If the per capita expenditure is increasing, either the numerator has to be increasing or the denominator has to be decreasing or both.

So, either the price is increasing, the number of pills is increasing, or the number of people is decreasing (or some combination of the above).

premise: product prices can't be raised (note: by definition, this only addresses existing products; new products not yet introduced do not yet have assigned prices)
--> So I can't raise the price of existing products, but I could introduce more expensive products.

premise: the use of prescription drugs did not increase after the price freeze
--> So the number of pills isn't changing
这道题由公式入手:人均药品消费量,如果提高说明分子变大,分母变小——只要答案里涉及其中一个即可

4.--->45s
(A) some prehistoric cave paintings in France also depict other animals as having a hump
(B) fossils of the giant deer are much more common in Ireland than in France
(C) animal humps are composed of fatty tissue, which does not fossilize
(D) the cave paintings of the giant deer were painted well before 16,000 years ago
(E) only one currently existing species of deer has any anatomical feature that even remotely resembles a hump
C

5.--->45s
A) Among the parts of its business that Vernon does not plan to outsource are some that require standards of accuracy too high for most independent suppliers to provide at lower cost than Vernon can.
(B) Vernon itself acts as an independent supplier of specialized hardware items to certain manufacturers that formerly made those items themselves.Vernon本身是某些制造商的独立供应商,这些制造商以前自己制造这些产品。跟profit半毛钱关系没有
(C) Relatively few manufacturers that start as independent suppliers have been able to expand their business and become direct competitors of the companies they once supplied.
(D) Vernon plans to select the independent suppliers it will use on the basis of submitted bids.
(E) Attending to certain tasks that Vernon performs relatively inefficiently has taken up much of the time and effort of top managers whose time would have been better spent attending to Vernon's core business.
B
OA:E
只有E是与boost profit(公司目的)有关的,其他都不是——这道题可能是因为没看懂选项……
作者: 云栈    时间: 2019-10-29 03:34
1.        32s
B: 人的五官被认为是不同的,互相之前都不会有影响。
P: 然而最近的研究发现有的人吃了香蕉说他们吃了蓝色;有的人看到一个颜色说它有某种气味。
C:这说明这种人的五官没有遵循明确的界限
推测:他们的感官没有互相刺激,只是错乱了;或者这个人的感官没错,只是他们表达出了错
选项分析:A
A:正确说明他们的无关没有问题,是他们的表达出了问题。
B:与原题无关,原题说的就是这五官
C:opposite,支持了原题的观点
D:无关,synesthetic被如何定义与五官的界限无关
E:opposite,吃了药之后就会不遵循界限

2.        27s
F: 毛虫在树上被孵化出来之后就会立刻往上爬,直到找到一个能吃的叶子
F:生物学家认为天性是他们朝着重力的反方向爬
F: 最近的实验把一个强光放在树底下,毛虫孵化出来之后往下爬
推测:毛虫爬的方向不一定和重力有关,和强光有关。
选项分析:C

3.        41s
B:人均每年在处方药上的花销增长了15%
F: 为了抑制这种增长,禁止了药厂增长他们产品的价格
F: 即使处方药的使用量没有增长,但是人均花销仍然上涨了
推测:买处方药的人变少了,但是买的人的用量增大了
选项分析:A
【这题没太看懂选项的逻辑,用排除法选的A

4.        39s
B:已经灭绝的欧洲夜鹿的最后一个成员叫giant,生活在16000年前的爱尔兰
P: 法国的历史cave画的这种鹿的背后有大的hump
P:然而这种动物的化石没有显示有hump
C: 然而没有理由说cave画的不准确
推测:变化石的过程中hump被腐蚀了之类的。
选项分析:C

5.        41s
P: outsourcing 是一种从其他独立供应商那里获得公司之前本身有的一种产品
P: V,一个小型公司最近收益下滑,计划通过outsourcing他们的部分商业给一个可以提供低成本的供应商来增长收益
推测:decline in profit的主要原因是成本太高了,外包可以给公司带来其他更大的收益
选项分析:E

作者: Lincy123    时间: 2020-4-27 23:33
P:研究发现有人吃香蕉说吃出了蓝色,或看到颜色说它有特殊味道
C:这类人S的感官不符合通常五感之间的范畴
果因推理,找他因
A) S有语言认知上的缺陷。Correct  
B) 五感之外还有其他感。无关
C) S的这类感官重叠情况有一种固定模式。增强
D)在多种文化中S被称为传奇。无关
E) 用毒品可以暂时摆脱S的情况。无关
幼虫孵化后往上爬
强光从树下往上打,幼虫孵化后往下爬
Must be true
A) 幼虫天生会朝着重力方向走。错
B) 幼虫在黑暗中看不见。错,看不见不会受光照影响
C) 幼虫倾向强光移动。Correct
D)幼虫朝着最近的叶子吃
E) 幼虫对光照不敏感。削弱
健康署禁止药厂涨价,使用数量不变
处方药的人均支出每年都在大幅度增长
反推:人均支出=总支出/人数,总支出升高或者人数减少
A) 制药商出新药替代旧药。Correct
B) 这个时期人口增长多。无关,因为使用量不变
C) 尽管价格不变,制造流程的改善让制药商有高利润。无关
D)政府鼓励医生开便宜的药。人均支出应减少。
E) 一些外国昂贵的制药商不在V营销了。应减少
鹿的化石没有隆起,不能说画家画了有隆起的鹿是错的
因为:?
可能有其他原因让hump的化石消失了
A) 这些画也画了其他动物有hump。无关
B) 爱尔兰的化石更常见。无关
C) Hump不能变成化石。Correct
D)壁画的年龄。无关
E) 只有一种鹿有hump。无关
P:V公司打算把部分业务外包给成本更低的公司
C:boost profit
Support:排除weaken
A) 没有外包的是外包做不到低成本的。无关
B) V是别家的外包公司,无关
C) 很少有供货商变成他们下游的直接竞争者。Correct
想太多了,变成竞争者和profit无关
D)V计划在提交的招标基础上选用独立供应商。无关
E) 参加V效率低的任务占据了高层很多参加核心业务的时间。无关
Correct 外包就能让高层把重心放在核心业务






欢迎光临 ChaseDream (https://forum.chasedream.com/) Powered by Discuz! X3.3